[obm-l] Re: [obm-l] Questão de probabilidade

2022-06-29 Por tôpico Rogerio Ponce
 cachorro
preto seria justamente 1/8. Entretanto, existe chance de que algum
outro cachorro tambem seja preto, de modo que a probabilidade de
escolhermos um animal ao acaso, e ele ser um cachorro preto, e' maior
que 1/8, e ja' sabemos que a afirmacao 08 esta' errada.


Mas, por curiosidade, vamos calcular a probabilidade de, escolhendo-se
um animal ao acaso, ele ser um cachorro preto.
Entao a probabilidade procurada e' igual 'a probabilidade de
escolhermos o cachorro que ja' seria preto somada 'a probabilidade de
escolhermos um dos outros animais, e ele ser preto, e ele ser um
cachorro.
Ou seja, a probabilidade pedida vale
[ 1/8 ] + [ 7/8 * 2/7 * 4/7 ] =  1/8 + 1/7 = 15/56

Uma outra forma de se pensar:

A probabilidade pedida e' a probabilidade de escolhermos um cachorro,
multiplicada pela probabilidade de ele ser preto.
Entre os 8 animais, a probabilidade de escolhermos um cachorro vale 5/8.

E a probabilidade de ele ser preto e' igual a soma da probabilidade de
ele ser o cachorro definido inicialmente como preto, somada 'a
probabilidade de ele ser um dos outros caes, e receber a pintura
preta.

Em sendo um cachorro, a probabilidade de ele ser o cao definido
inicialmente como preto vale 1/5.
E a probabilidade deste cachorro ser um dos outros caes vale 4/5, e,
neste caso, a probabilidade deste animal receber a pintura preta vale
2/7, pois temos 2 pinturas pretas entre um total de 7 animais.

Entao, a probabilidade pedida vale:
5/8 *  (1/5 + 4/5 * 2/7) = 5/8 * (1/5 + 8/35) = 1/8 + 1/7 = 15/56

De modo que a afirmacao 08 esta' errada.



Afirmacao 16) Se um animal for escolhido ao acaso, a probabilidade de
ele ser um gato malhado e' de 1/16.

-

Entre as 8 pinturas, a probabilidade de pegarmos o animal com a
pintura malhada e' 1/8. E a probabilidade deste animal ser um gato
(lembrando que um dos caes ja' e' preto) e' 3/7.

Assim, a probabilidade de escolhermos um gato malhado vale (1/8) * (3/7) = 3/56.

Portanto, a afirmacao 16 esta' errada.



Ao final de tudo, vemos que a unica afirmacao correta e' a 04.

[]'s
Rogerio Ponce


===





 54,1 Topo

On Wed, Mar 16, 2022 at 8:08 AM Professor Vanderlei Nemitz
 wrote:
>
> Bom dia!
> Na questão a seguir, do vestibular da UEM, penso que o espaço amostral tem 
> 105 elementos, pois um cachorro é preto (desconsideramos esse). Porém, com 
> esse pensamento, não consigo obter o gabarito, que diz que 02 e 16 são 
> corretas.
> Alguém poderia ajudar?
> Muito obrigado!
>
> Em um pet shop há 3 gatos e 5 cães. Sabemos que 3 desses animais são pretos, 
> 4 são brancos e 1 é malhado. Além disso, pelo menos 1 cachorro é preto. 
> Assinale o que for correto.
> 01) A probabilidade de haver exatamente 1 cachorro preto é de 1/6.
> 02) A probabilidade de haver pelo menos 1 gato branco e pelo menos 2 
> cachorros brancos é de 2/3.
> 04) A probabilidade de haver um cachorro malhado é maior do que a 
> probabilidade de haver um gato malhado.
> 08) Se um animal for escolhido ao acaso, a probabilidade de ele ser um 
> cachorro preto é de 1/8.
> 16) Se um animal for escolhido ao acaso, a probabilidade de ele ser um gato 
> malhado é de 1/16.
>
> --
> Esta mensagem foi verificada pelo sistema de antivírus e
> acredita-se estar livre de perigo.

-- 
Esta mensagem foi verificada pelo sistema de antiv�rus e
 acredita-se estar livre de perigo.


=
Instru��es para entrar na lista, sair da lista e usar a lista em
http://www.mat.puc-rio.br/~obmlistas/obm-l.html
=


[obm-l] Re: [obm-l] Re: [obm-l] Re: [obm-l] Re: [obm-l] Re: [obm-l] Questão de probabilidade

2022-06-24 Por tôpico Rogerio Ponce
Otima explicacao!
Obrigado, Ralph!

PS: e sim, a provocacao foi pra voce mesmo!
:)

[]'s
Rogerio Ponce


On Wed, Jun 22, 2022 at 1:00 PM Ralph Costa Teixeira  wrote:
>
> Ponce está provocando a gente... senti que esta flecha tinha um bocado a 
> minha direção...  :D :D :D
>
> Olha, tem duas "visões" sobre o que "probabilidade" significa.
>
> A primeira vai na linha de que só podemos falar de probabilidade sobre coisas 
> que ainda não aconteceram. Vai nessa linha: se os eventos estão no passado, 
> então já aconteceram, já estão definidos, e não faz sentido dizer que tinha x 
> de chance de ser assim ou y de ser assado. Se você já jogou a moeda justa, 
> não é mais 50/50 --  é 100% de ser cara, ou 100% de ser coroa, dependendo do 
> que ocorreu. Quem pensa assim vai dizer que dado um certo evento (sempre no 
> futuro), ele tem uma probabilidade dada; se duas pessoas diferentes derem 
> duas probabilidades diferentes para o mesmo evento, uma delas errou.
>
> Outra linha diz que podemos falar de probabilidade sempre que houver 
> incerteza; não interessa o que aconteceu ou o que vai acontecer, o que 
> interessa é o que você SABE sobre o acontecimento. Se você jogou a moeda 
> justa mas eu não sei nada mais sobre o lançamento, continua sendo 50/50 
> **PARA MIM**. Probabilidade passa a ser um conceito sobre INFORMAÇÃO, não 
> sobre os fatos em si (a probabilidade não está na moeda, está no que você 
> sabe sobre a moeda). Quem pensa assim vai dizer que a probabilidade do evento 
> depende não apenas do evento em si, mas da informação que se tem em mãos. 
> Quem pensa assim admite que duas pessoas diferentes podem dar probabilidades 
> diferentes ao mesmo evento SE SOUBEREM FATOS DIFERENTES a respeito do evento, 
> ou seja, probabilidade passa a ser bastante "subjetivo".
>
> Eu talvez tenha descrito mal a primeira interpretação, pois sou ferrenho 
> defensor da segunda. Ela simplesmente engloba a primeira, porque você pode 
> ter informação parcial sobre fatos que ocorrem no futuro. E falar de 
> probabilidade para descrever incerteza presente ou passada é MUITO útil! Eu 
> quero poder expressar incerteza sobre fatos passados com frases do tipo 
> "fulano tem x% de probabilidade de ter cometido tal crime", ou "tem y% de 
> probabilidade de ter petróleo nesse poço", ou "tem z% de chance de eu ter 
> COVID"... Se você tem uma reação negativa a essas frases, lembre o que elas 
> realmente significam (na segunda interpretação): claro que ou o cara cometeu 
> o crime ou não, não faz sentido dizer que ele cometeu o crime x% das vezes em 
> que fizermos um experimento de ele cometer o crime... mas o que aquilo 
> significa é "com a informação que eu tenho, numa escala de 0 a 1, eu tenho 
> x/100 de certeza que fulano cometeu o crime". E "certeza baseada em informaç�!
 �es" é sim quantificável -- e satisfaz exatamente as leis das probabilidades 
com as quais concordamos. "Subjetivo" não significa "posso falar qualquer 
coisa", significa apenas que a conta pode variar de pessoa para pessoa... mas, 
de novo, SE ESSAS PESSOAS TIVEREM INFORMAÇÕES DIFERENTES sobre o evento.
>
> Abraço, Ralph.
>
> On Wed, Jun 22, 2022 at 12:09 PM Rogerio Ponce  wrote:
>>
>> Olá Pedro e pessoal da lista!
>>
>> Segundo a opinião do Pedro, nao faz sentido perguntar qual a probabilidade 
>> de Jose ter conseguido um 6 ao jogar o dado ontem, pois isso ja' aconteceu, 
>> e, portanto, ja' esta' definido.
>>
>> Sera' que e' isso mesmo?
>>
>> []'s
>> Rogerio Ponce
>>
>>
>> On Mon, Jun 20, 2022 at 9:45 PM Pedro José  wrote:
>>>
>>> Eu na minha humilde opinião creio que a probabilidade exista quando pode 
>>> ser uma coisa ou outra. No caso já é definido o que os animais são. Então 
>>> já está tudo errado. A questão seria viável se dessem esses limitantes para 
>>> uma criança que pintaria os desenhos dos animais. Aí sim há probabilidade.
>>>
>>> Em sáb., 18 de jun. de 2022 03:33, Rogerio Ponce  
>>> escreveu:
>>>>
>>>> Ola' Vanderlei e pessoal da lista!
>>>>
>>>> Sem perda de generalidade, podemos imaginar que vamos fazer o seguinte:
>>>>
>>>> - uma pintura preta em um dos caes, escolhido aleatoriamente
>>>>
>>>> - uma pintura "malhada" em um dos animais, escolhido aleatoriamente entre 
>>>> os 7 animais nao pintados
>>>>
>>>> - duas pintura pretas, em dois animais, escolhidos aleatoriamente entre os 
>>>> 6 animais restantes,
>>>>
>>>> - quatro pinturas brancas nos 4 animais restantes
>&

[obm-l] Re: [obm-l] Re: [obm-l] Re: [obm-l] Re: [obm-l] Questão de probabilidade

2022-06-22 Por tôpico Ralph Costa Teixeira
Ponce está provocando a gente... senti que esta flecha tinha um bocado a
minha direção...  :D :D :D

Olha, tem duas "visões" sobre o que "probabilidade" significa.

A primeira vai na linha de que só podemos falar de probabilidade sobre
coisas que ainda não aconteceram. Vai nessa linha: se os eventos estão no
passado, então já aconteceram, já estão definidos, e não faz sentido dizer
que tinha x de chance de ser assim ou y de ser assado. Se você já jogou a
moeda justa, não é mais 50/50 --  é 100% de ser cara, ou 100% de ser coroa,
dependendo do que ocorreu. Quem pensa assim vai dizer que dado um certo
evento (sempre no futuro), ele tem uma probabilidade dada; se duas pessoas
diferentes derem duas probabilidades diferentes para o mesmo evento, uma
delas errou.

Outra linha diz que podemos falar de probabilidade sempre que houver
incerteza; não interessa o que aconteceu ou o que vai acontecer, o que
interessa é o que você SABE sobre o acontecimento. Se você jogou a moeda
justa mas eu não sei nada mais sobre o lançamento, continua sendo 50/50
**PARA MIM**. Probabilidade passa a ser um conceito sobre INFORMAÇÃO, não
sobre os fatos em si (a probabilidade não está na moeda, está no que você
sabe sobre a moeda). Quem pensa assim vai dizer que a probabilidade do
evento depende não apenas do evento em si, mas da informação que se tem em
mãos. Quem pensa assim admite que duas pessoas diferentes podem dar
probabilidades diferentes ao mesmo evento SE SOUBEREM FATOS DIFERENTES a
respeito do evento, ou seja, probabilidade passa a ser bastante "subjetivo".

Eu talvez tenha descrito mal a primeira interpretação, pois sou ferrenho
defensor da segunda. Ela simplesmente engloba a primeira, porque você pode
ter informação parcial sobre fatos que ocorrem no futuro. E falar de
probabilidade para descrever incerteza presente ou passada é MUITO útil! Eu
quero poder expressar incerteza sobre fatos passados com frases do tipo
"fulano tem x% de probabilidade de ter cometido tal crime", ou "tem y% de
probabilidade de ter petróleo nesse poço", ou "tem z% de chance de eu ter
COVID"... Se você tem uma reação negativa a essas frases, lembre o que elas
realmente significam (na segunda interpretação): claro que ou o cara
cometeu o crime ou não, não faz sentido dizer que ele cometeu o crime x%
das vezes em que fizermos um experimento de ele cometer o crime... mas o
que aquilo significa é "com a informação que eu tenho, numa escala de 0 a
1, eu tenho x/100 de certeza que fulano cometeu o crime". E "certeza
baseada em informações" é sim quantificável -- e satisfaz exatamente as
leis das probabilidades com as quais concordamos. "Subjetivo" não significa
"posso falar qualquer coisa", significa apenas que a conta pode variar de
pessoa para pessoa... mas, de novo, SE ESSAS PESSOAS TIVEREM INFORMAÇÕES
DIFERENTES sobre o evento.

Abraço, Ralph.

On Wed, Jun 22, 2022 at 12:09 PM Rogerio Ponce  wrote:

> Olá Pedro e pessoal da lista!
>
> Segundo a opinião do Pedro, nao faz sentido perguntar qual a probabilidade
> de Jose ter conseguido um 6 ao jogar o dado ontem, pois isso ja' aconteceu,
> e, portanto, ja' esta' definido.
>
> Sera' que e' isso mesmo?
>
> []'s
> Rogerio Ponce
>
>
> On Mon, Jun 20, 2022 at 9:45 PM Pedro José  wrote:
>
>> Eu na minha humilde opinião creio que a probabilidade exista quando pode
>> ser uma coisa ou outra. No caso já é definido o que os animais são. Então
>> já está tudo errado. A questão seria viável se dessem esses limitantes para
>> uma criança que pintaria os desenhos dos animais. Aí sim há probabilidade.
>>
>> Em sáb., 18 de jun. de 2022 03:33, Rogerio Ponce da Silva <
>> abrlw...@gmail.com> escreveu:
>>
>>> Ola' Vanderlei e pessoal da lista!
>>>
>>> Sem perda de generalidade, podemos imaginar que vamos fazer o seguinte:
>>>
>>> - uma pintura preta em um dos caes, escolhido aleatoriamente
>>>
>>> - uma pintura "malhada" em um dos animais, escolhido aleatoriamente
>>> entre os 7 animais nao pintados
>>>
>>> - duas pintura pretas, em dois animais, escolhidos aleatoriamente entre
>>> os 6 animais restantes,
>>>
>>> - quatro pinturas brancas nos 4 animais restantes
>>>
>>>
>>> Analisando a afirmacao 04, por exemplo, verificamos que, no segundo
>>> passo (pintura malhada) existem 4 opcoes de cachorro e 3 opcoes de gato.
>>>
>>> Assim, a probabilidade de haver um cachorro malhado (4/7) e' maior que a
>>> probabilidade de haver um gato malhado (3/7).
>>> Portanto, a afirmacao 04 esta' correta.
>>> (e o gabarito esta' errado).
>>>
>>> []'s
>>> Rogerio Ponce
>>>
>>>
>>>
>

[obm-l] Re: [obm-l] Re: [obm-l] Re: [obm-l] Questão de probabilidade

2022-06-22 Por tôpico Rogerio Ponce
Olá Pedro e pessoal da lista!

Segundo a opinião do Pedro, nao faz sentido perguntar qual a probabilidade
de Jose ter conseguido um 6 ao jogar o dado ontem, pois isso ja' aconteceu,
e, portanto, ja' esta' definido.

Sera' que e' isso mesmo?

[]'s
Rogerio Ponce


On Mon, Jun 20, 2022 at 9:45 PM Pedro José  wrote:

> Eu na minha humilde opinião creio que a probabilidade exista quando pode
> ser uma coisa ou outra. No caso já é definido o que os animais são. Então
> já está tudo errado. A questão seria viável se dessem esses limitantes para
> uma criança que pintaria os desenhos dos animais. Aí sim há probabilidade.
>
> Em sáb., 18 de jun. de 2022 03:33, Rogerio Ponce da Silva <
> abrlw...@gmail.com> escreveu:
>
>> Ola' Vanderlei e pessoal da lista!
>>
>> Sem perda de generalidade, podemos imaginar que vamos fazer o seguinte:
>>
>> - uma pintura preta em um dos caes, escolhido aleatoriamente
>>
>> - uma pintura "malhada" em um dos animais, escolhido aleatoriamente entre
>> os 7 animais nao pintados
>>
>> - duas pintura pretas, em dois animais, escolhidos aleatoriamente entre
>> os 6 animais restantes,
>>
>> - quatro pinturas brancas nos 4 animais restantes
>>
>>
>> Analisando a afirmacao 04, por exemplo, verificamos que, no segundo passo
>> (pintura malhada) existem 4 opcoes de cachorro e 3 opcoes de gato.
>>
>> Assim, a probabilidade de haver um cachorro malhado (4/7) e' maior que a
>> probabilidade de haver um gato malhado (3/7).
>> Portanto, a afirmacao 04 esta' correta.
>> (e o gabarito esta' errado).
>>
>> []'s
>> Rogerio Ponce
>>
>>
>>
>> On Wed, Mar 16, 2022 at 8:08 AM Professor Vanderlei Nemitz <
>> vanderma...@gmail.com> wrote:
>>
>>> Bom dia!
>>> Na questão a seguir, do vestibular da UEM, penso que o espaço amostral
>>> tem 105 elementos, pois um cachorro é preto (desconsideramos esse). Porém,
>>> com esse pensamento, não consigo obter o gabarito, que diz que 02 e 16 são
>>> corretas.
>>> Alguém poderia ajudar?
>>> Muito obrigado!
>>>
>>> *Em um pet shop há 3 gatos e 5 cães. Sabemos que 3 desses animais são
>>> pretos, 4 são brancos e 1 é malhado. Além disso, pelo menos 1 cachorro é
>>> preto. Assinale o que for correto. *
>>> *01) A probabilidade de haver exatamente 1 cachorro preto é de 1/6. *
>>> *02) A probabilidade de haver pelo menos 1 gato branco e pelo menos 2
>>> cachorros brancos é de 2/3.*
>>> *04) A probabilidade de haver um cachorro malhado é maior do que a
>>> probabilidade de haver um gato malhado. *
>>> *08) Se um animal for escolhido ao acaso, a probabilidade de ele ser um
>>> cachorro preto é de 1/8. *
>>> *16) Se um animal for escolhido ao acaso, a probabilidade de ele ser um
>>> gato malhado é de 1/16.   *
>>>
>>> --
>>> Esta mensagem foi verificada pelo sistema de antivírus e
>>> acredita-se estar livre de perigo.
>>
>>
>> --
>> Esta mensagem foi verificada pelo sistema de antivírus e
>> acredita-se estar livre de perigo.
>
>
> --
> Esta mensagem foi verificada pelo sistema de antivírus e
> acredita-se estar livre de perigo.

-- 
Esta mensagem foi verificada pelo sistema de antiv�rus e
 acredita-se estar livre de perigo.



[obm-l] Re: [obm-l] Re: [obm-l] Questão de probabilidade

2022-06-20 Por tôpico Pedro José
Eu na minha humilde opinião creio que a probabilidade exista quando pode
ser uma coisa ou outra. No caso já é definido o que os animais são. Então
já está tudo errado. A questão seria viável se dessem esses limitantes para
uma criança que pintaria os desenhos dos animais. Aí sim há probabilidade.

Em sáb., 18 de jun. de 2022 03:33, Rogerio Ponce da Silva <
abrlw...@gmail.com> escreveu:

> Ola' Vanderlei e pessoal da lista!
>
> Sem perda de generalidade, podemos imaginar que vamos fazer o seguinte:
>
> - uma pintura preta em um dos caes, escolhido aleatoriamente
>
> - uma pintura "malhada" em um dos animais, escolhido aleatoriamente entre
> os 7 animais nao pintados
>
> - duas pintura pretas, em dois animais, escolhidos aleatoriamente entre os
> 6 animais restantes,
>
> - quatro pinturas brancas nos 4 animais restantes
>
>
> Analisando a afirmacao 04, por exemplo, verificamos que, no segundo passo
> (pintura malhada) existem 4 opcoes de cachorro e 3 opcoes de gato.
>
> Assim, a probabilidade de haver um cachorro malhado (4/7) e' maior que a
> probabilidade de haver um gato malhado (3/7).
> Portanto, a afirmacao 04 esta' correta.
> (e o gabarito esta' errado).
>
> []'s
> Rogerio Ponce
>
>
>
> On Wed, Mar 16, 2022 at 8:08 AM Professor Vanderlei Nemitz <
> vanderma...@gmail.com> wrote:
>
>> Bom dia!
>> Na questão a seguir, do vestibular da UEM, penso que o espaço amostral
>> tem 105 elementos, pois um cachorro é preto (desconsideramos esse). Porém,
>> com esse pensamento, não consigo obter o gabarito, que diz que 02 e 16 são
>> corretas.
>> Alguém poderia ajudar?
>> Muito obrigado!
>>
>> *Em um pet shop há 3 gatos e 5 cães. Sabemos que 3 desses animais são
>> pretos, 4 são brancos e 1 é malhado. Além disso, pelo menos 1 cachorro é
>> preto. Assinale o que for correto. *
>> *01) A probabilidade de haver exatamente 1 cachorro preto é de 1/6. *
>> *02) A probabilidade de haver pelo menos 1 gato branco e pelo menos 2
>> cachorros brancos é de 2/3.*
>> *04) A probabilidade de haver um cachorro malhado é maior do que a
>> probabilidade de haver um gato malhado. *
>> *08) Se um animal for escolhido ao acaso, a probabilidade de ele ser um
>> cachorro preto é de 1/8. *
>> *16) Se um animal for escolhido ao acaso, a probabilidade de ele ser um
>> gato malhado é de 1/16.   *
>>
>> --
>> Esta mensagem foi verificada pelo sistema de antivírus e
>> acredita-se estar livre de perigo.
>
>
> --
> Esta mensagem foi verificada pelo sistema de antivírus e
> acredita-se estar livre de perigo.

-- 
Esta mensagem foi verificada pelo sistema de antiv�rus e
 acredita-se estar livre de perigo.



[obm-l] Re: [obm-l] Questão de probabilidade

2022-06-18 Por tôpico Rogerio Ponce da Silva
Ola' Vanderlei e pessoal da lista!

Sem perda de generalidade, podemos imaginar que vamos fazer o seguinte:

- uma pintura preta em um dos caes, escolhido aleatoriamente

- uma pintura "malhada" em um dos animais, escolhido aleatoriamente entre
os 7 animais nao pintados

- duas pintura pretas, em dois animais, escolhidos aleatoriamente entre os
6 animais restantes,

- quatro pinturas brancas nos 4 animais restantes


Analisando a afirmacao 04, por exemplo, verificamos que, no segundo passo
(pintura malhada) existem 4 opcoes de cachorro e 3 opcoes de gato.

Assim, a probabilidade de haver um cachorro malhado (4/7) e' maior que a
probabilidade de haver um gato malhado (3/7).
Portanto, a afirmacao 04 esta' correta.
(e o gabarito esta' errado).

[]'s
Rogerio Ponce



On Wed, Mar 16, 2022 at 8:08 AM Professor Vanderlei Nemitz <
vanderma...@gmail.com> wrote:

> Bom dia!
> Na questão a seguir, do vestibular da UEM, penso que o espaço amostral tem
> 105 elementos, pois um cachorro é preto (desconsideramos esse). Porém, com
> esse pensamento, não consigo obter o gabarito, que diz que 02 e 16 são
> corretas.
> Alguém poderia ajudar?
> Muito obrigado!
>
> *Em um pet shop há 3 gatos e 5 cães. Sabemos que 3 desses animais são
> pretos, 4 são brancos e 1 é malhado. Além disso, pelo menos 1 cachorro é
> preto. Assinale o que for correto. *
> *01) A probabilidade de haver exatamente 1 cachorro preto é de 1/6. *
> *02) A probabilidade de haver pelo menos 1 gato branco e pelo menos 2
> cachorros brancos é de 2/3.*
> *04) A probabilidade de haver um cachorro malhado é maior do que a
> probabilidade de haver um gato malhado. *
> *08) Se um animal for escolhido ao acaso, a probabilidade de ele ser um
> cachorro preto é de 1/8. *
> *16) Se um animal for escolhido ao acaso, a probabilidade de ele ser um
> gato malhado é de 1/16.   *
>
> --
> Esta mensagem foi verificada pelo sistema de antivírus e
> acredita-se estar livre de perigo.

-- 
Esta mensagem foi verificada pelo sistema de antiv�rus e
 acredita-se estar livre de perigo.



[obm-l] Re: Questão de probabilidade

2022-03-16 Por tôpico Professor Vanderlei Nemitz
Hummm...
Acho que descobri o que o autor pensou.
Parece que 6 casos, distribuindo P, P, B, B, B, B, M (2 pretos, 4 brancos e
1 malhado) entre 3 gatos e 4 cachorros.
Mas...esse espaco amostral é equiprovável???

Em qua., 16 de mar. de 2022 07:58, Professor Vanderlei Nemitz <
vanderma...@gmail.com> escreveu:

> Bom dia!
> Na questão a seguir, do vestibular da UEM, penso que o espaço amostral tem
> 105 elementos, pois um cachorro é preto (desconsideramos esse). Porém, com
> esse pensamento, não consigo obter o gabarito, que diz que 02 e 16 são
> corretas.
> Alguém poderia ajudar?
> Muito obrigado!
>
> *Em um pet shop há 3 gatos e 5 cães. Sabemos que 3 desses animais são
> pretos, 4 são brancos e 1 é malhado. Além disso, pelo menos 1 cachorro é
> preto. Assinale o que for correto. *
> *01) A probabilidade de haver exatamente 1 cachorro preto é de 1/6. *
> *02) A probabilidade de haver pelo menos 1 gato branco e pelo menos 2
> cachorros brancos é de 2/3.*
> *04) A probabilidade de haver um cachorro malhado é maior do que a
> probabilidade de haver um gato malhado. *
> *08) Se um animal for escolhido ao acaso, a probabilidade de ele ser um
> cachorro preto é de 1/8. *
> *16) Se um animal for escolhido ao acaso, a probabilidade de ele ser um
> gato malhado é de 1/16.   *
>

-- 
Esta mensagem foi verificada pelo sistema de antiv�rus e
 acredita-se estar livre de perigo.



[obm-l] Questão de probabilidade

2022-03-16 Por tôpico Professor Vanderlei Nemitz
Bom dia!
Na questão a seguir, do vestibular da UEM, penso que o espaço amostral tem
105 elementos, pois um cachorro é preto (desconsideramos esse). Porém, com
esse pensamento, não consigo obter o gabarito, que diz que 02 e 16 são
corretas.
Alguém poderia ajudar?
Muito obrigado!

*Em um pet shop há 3 gatos e 5 cães. Sabemos que 3 desses animais são
pretos, 4 são brancos e 1 é malhado. Além disso, pelo menos 1 cachorro é
preto. Assinale o que for correto. *
*01) A probabilidade de haver exatamente 1 cachorro preto é de 1/6. *
*02) A probabilidade de haver pelo menos 1 gato branco e pelo menos 2
cachorros brancos é de 2/3.*
*04) A probabilidade de haver um cachorro malhado é maior do que a
probabilidade de haver um gato malhado. *
*08) Se um animal for escolhido ao acaso, a probabilidade de ele ser um
cachorro preto é de 1/8. *
*16) Se um animal for escolhido ao acaso, a probabilidade de ele ser um
gato malhado é de 1/16.   *

-- 
Esta mensagem foi verificada pelo sistema de antiv�rus e
 acredita-se estar livre de perigo.



[obm-l] Re: [obm-l] Ajuda numa questão da OBM 1987

2021-08-21 Por tôpico Anderson Torres
Em ter., 20 de jul. de 2021 às 18:25, Prof. Douglas Oliveira
 escreveu:
>
> Tem-se um bolo em forma de prisma triangular, cuja base está em um plano 
> horizontal. Dois indivíduos vão dividir o bolo de acordo com a seguinte 
> regra: o primeiro escolhe um ponto na base superior do bolo e o segundo corta 
> o bolo por um plano vertical à sua escolha, passando porém pelo ponto 
> escolhido, e seleciona para si um dos pedaços em que dividiu o bolo. Qual 
> deve ser a estratégia para o primeiro e qual deve ser a fração do volume do 
> bolo que ele espera obter?

Primeira dica: tente resolver o mesmo problema, mas para um triângulo
equilátero de lado 100.
Afinal de contas, uma transformação afim leva isso para qualquer triângulo.

>
> Abraço do Douglas.
>
> --
> Esta mensagem foi verificada pelo sistema de antivírus e
> acredita-se estar livre de perigo.

-- 
Esta mensagem foi verificada pelo sistema de antiv�rus e
 acredita-se estar livre de perigo.


=
Instru��es para entrar na lista, sair da lista e usar a lista em
http://www.mat.puc-rio.br/~obmlistas/obm-l.html
=


[obm-l] Re: [obm-l] Ajuda numa questão da OBM 1987

2021-07-20 Por tôpico joao pedro b menezes
Eu pensaria em trabalhar com os pontos notáveis, talvez o baricentro, e
argumentar que em qualquer outro ponto é possível realizar um corte que o
prejudique mais. Isso é só uma teoria e, portanto, é possível que esteja
totalmente errada.

-- 
Esta mensagem foi verificada pelo sistema de antiv�rus e
 acredita-se estar livre de perigo.



[obm-l] Ajuda numa questão da OBM 1987

2021-07-20 Por tôpico Prof. Douglas Oliveira
*Tem-se um bolo em forma de prisma triangular, cuja base está em um plano
horizontal. Dois indivíduos vão dividir o bolo de acordo com a seguinte
regra: o primeiro escolhe um ponto na base superior do bolo e o segundo
corta o bolo por um plano vertical à sua escolha, passando porém pelo ponto
escolhido, e seleciona para si um dos pedaços em que dividiu o bolo.
Qual deve ser a estratégia para o primeiro e qual deve ser a fração do
volume do bolo que ele espera obter?*

*Abraço do Douglas.*

-- 
Esta mensagem foi verificada pelo sistema de antiv�rus e
 acredita-se estar livre de perigo.



[obm-l] Re: [obm-l] Re: [obm-l] Questão sobre desigualdades

2021-04-14 Por tôpico Carlos Monteiro
De onde saiu essa desigualdade?

Em qua., 14 de abr. de 2021 às 20:39, Anderson Torres <
torres.anderson...@gmail.com> escreveu:

> Em qua., 14 de abr. de 2021 às 15:54, Carlos Monteiro
>  escreveu:
> >
> > Encontre os valores máximo e mínimo da expressão:  x/(x^2+1) + y/(y^2+1)
> + z/(z^2+1) , onde x, y e z são números reais que satisfazem x+y+z = 1.
> >
> >
>
> Verifica-se que 3(12x+1)/50 >= x/(x^2+1), e assim o valor máximo é 3/10
>
> >
> >
> > --
> > Esta mensagem foi verificada pelo sistema de antivírus e
> > acredita-se estar livre de perigo.
>
> --
> Esta mensagem foi verificada pelo sistema de antivírus e
>  acredita-se estar livre de perigo.
>
>
> =
> Instru�ões para entrar na lista, sair da lista e usar a lista em
> http://www.mat.puc-rio.br/~obmlistas/obm-l.html
> =
>

-- 
Esta mensagem foi verificada pelo sistema de antiv�rus e
 acredita-se estar livre de perigo.



[obm-l] Re: [obm-l] Questão sobre desigualdades

2021-04-14 Por tôpico Anderson Torres
Em qua., 14 de abr. de 2021 às 15:54, Carlos Monteiro
 escreveu:
>
> Encontre os valores máximo e mínimo da expressão:  x/(x^2+1) + y/(y^2+1) + 
> z/(z^2+1) , onde x, y e z são números reais que satisfazem x+y+z = 1.
>
>

Verifica-se que 3(12x+1)/50 >= x/(x^2+1), e assim o valor máximo é 3/10

>
>
> --
> Esta mensagem foi verificada pelo sistema de antivírus e
> acredita-se estar livre de perigo.

-- 
Esta mensagem foi verificada pelo sistema de antiv�rus e
 acredita-se estar livre de perigo.


=
Instru��es para entrar na lista, sair da lista e usar a lista em
http://www.mat.puc-rio.br/~obmlistas/obm-l.html
=


[obm-l] Questão sobre desigualdades

2021-04-14 Por tôpico Carlos Monteiro
Encontre os valores máximo e mínimo da expressão:  x/(x^2+1) + y/(y^2+1) +
z/(z^2+1) , onde x, y e z são números reais que satisfazem x+y+z = 1.

-- 
Esta mensagem foi verificada pelo sistema de antiv�rus e
 acredita-se estar livre de perigo.



[obm-l] Re: [obm-l] Re: [obm-l] Re: [obm-l] Questão OBM - U

2020-01-23 Por tôpico Ralph Teixeira
Seja ABCD o quadrilatero convexo, e seja P o encontro das diagonais.

No triangulo APB, temos AP+PB>AB. Escreva as desigualdades analogas para os
triangulos BPC, CPD e DPA. Somando-as, voce vai obter que

2(AC+BD)>perimetro=8

Ou seja, o infimo tem que ser pelo menos 4.

Agora, para chegar no infimo, voce vai ter que "degenerar" os triangulos...
Entao considere um quadrilatero do tipo ABCB (ou seja, tome D=B), com,
digamos, AC=BC=2. Note que o perimetro eh 8, enquando AC=4 e BB=0, ou seja,
a soma das diagonais eh 4.

Mas alguns diriam que isso nao eh um quadrilatero convexo (bom, depende da
sua definicao de quadrilatero!)... Entao se "quadrilateros" nao incluem
casos degenrados, para fazer isso ficar rigoroso, voce teria que tomar um
quadrilatero convexo QUASE degenerado de perimetro 8 (um losango serve, a
conta fica facil), e mostrar que a soma das diagonais fica tao perto de 4
quanto voce queira.

Abraco, Ralph.

On Thu, Jan 23, 2020 at 7:24 AM gilberto azevedo 
wrote:

> Tentei com o retângulo e o quadrado, porém não obtive a resposta...  O
> gabarito é 4.
>
> Em sáb, 11 de jan de 2020 12:03, Bernardo Freitas Paulo da Costa <
> bernardo...@gmail.com> escreveu:
>
>> On Sat, Jan 11, 2020 at 11:24 AM gilberto azevedo 
>> wrote:
>> >
>> > Qual o ínfimo sobre todos os quadriláteros convexos com perímetro 8
>> da soma dos comprimentos de suas diagonais ?
>>
>> Quais são os quadriláteros que você tentaria?
>> --
>> Bernardo Freitas Paulo da Costa
>>
>> --
>> Esta mensagem foi verificada pelo sistema de antivírus e
>>  acredita-se estar livre de perigo.
>>
>>
>> =
>> Instruções para entrar na lista, sair da lista e usar a lista em
>> http://www.mat.puc-rio.br/~obmlistas/obm-l.html
>> =
>>
>
> --
> Esta mensagem foi verificada pelo sistema de antivírus e
> acredita-se estar livre de perigo.

-- 
Esta mensagem foi verificada pelo sistema de antiv�rus e
 acredita-se estar livre de perigo.



[obm-l] Re: [obm-l] Re: [obm-l] Re: [obm-l] Re: [obm-l] Re: [obm-l] Re: [obm-l] Questão OBM - U

2020-01-23 Por tôpico Esdras Muniz
É fácil ver que esse ínfimo tem que ser no mínimo 4, basta fazer
desigualdade triângulos com os triângulos que têm dois vértices comuns com
o quadrilátero e o terceiro sendo a interseção das diagonais. E por esse
argumento do Caio, vemos que é 4 mesmo.

Em qui, 23 de jan de 2020 08:59, Caio Costa  escreveu:

> Minimiza-se a soma das diagonais ao tomar-se um losango degenerado, com
> uma diagonal valendo 4 e outra valendo 0.
>
> Em qui, 23 de jan de 2020 08:34, gilberto azevedo 
> escreveu:
>
>> Pensei em minimizar √(a² + (4-a)²)
>> 4 - a, devido ao fato do perímetro ser 8.
>> No caso obtenho o mínimo sendo 2√2, quando o retângulo é um quadrado de
>> lado 2.
>> A soma das diagonais seria no caso 4√2, e não bate com o gabarito.
>>
>> Em qui, 23 de jan de 2020 08:20, Bernardo Freitas Paulo da Costa <
>> bernardo...@gmail.com> escreveu:
>>
>>> On Thu, Jan 23, 2020 at 7:24 AM gilberto azevedo 
>>> wrote:
>>> >> On Sat, Jan 11, 2020 at 11:24 AM gilberto azevedo <
>>> gil159...@gmail.com> wrote:
>>> >> >
>>> >> > Qual o ínfimo sobre todos os quadriláteros convexos com
>>> perímetro 8 da soma dos comprimentos de suas diagonais ?
>>> >
>>> > Tentei com o retângulo e o quadrado, porém não obtive a resposta...  O
>>> gabarito é 4.
>>>
>>> Qual (ou quais?) retângulo(s) você testou??  Que resposta você obteve?
>>> --
>>> Bernardo Freitas Paulo da Costa
>>>
>>> --
>>> Esta mensagem foi verificada pelo sistema de antivírus e
>>>  acredita-se estar livre de perigo.
>>>
>>>
>>> =
>>> Instru�ões para entrar na lista, sair da lista e usar a lista em
>>> http://www.mat.puc-rio.br/~obmlistas/obm-l.html
>>> =
>>>
>>
>> --
>> Esta mensagem foi verificada pelo sistema de antivírus e
>> acredita-se estar livre de perigo.
>
>
> --
> Esta mensagem foi verificada pelo sistema de antivírus e
> acredita-se estar livre de perigo.

-- 
Esta mensagem foi verificada pelo sistema de antiv�rus e
 acredita-se estar livre de perigo.



[obm-l] Re: [obm-l] Re: [obm-l] Re: [obm-l] Re: [obm-l] Re: [obm-l] Questão OBM - U

2020-01-23 Por tôpico Caio Costa
Minimiza-se a soma das diagonais ao tomar-se um losango degenerado, com uma
diagonal valendo 4 e outra valendo 0.

Em qui, 23 de jan de 2020 08:34, gilberto azevedo 
escreveu:

> Pensei em minimizar √(a² + (4-a)²)
> 4 - a, devido ao fato do perímetro ser 8.
> No caso obtenho o mínimo sendo 2√2, quando o retângulo é um quadrado de
> lado 2.
> A soma das diagonais seria no caso 4√2, e não bate com o gabarito.
>
> Em qui, 23 de jan de 2020 08:20, Bernardo Freitas Paulo da Costa <
> bernardo...@gmail.com> escreveu:
>
>> On Thu, Jan 23, 2020 at 7:24 AM gilberto azevedo 
>> wrote:
>> >> On Sat, Jan 11, 2020 at 11:24 AM gilberto azevedo 
>> wrote:
>> >> >
>> >> > Qual o ínfimo sobre todos os quadriláteros convexos com perímetro
>> 8 da soma dos comprimentos de suas diagonais ?
>> >
>> > Tentei com o retângulo e o quadrado, porém não obtive a resposta...  O
>> gabarito é 4.
>>
>> Qual (ou quais?) retângulo(s) você testou??  Que resposta você obteve?
>> --
>> Bernardo Freitas Paulo da Costa
>>
>> --
>> Esta mensagem foi verificada pelo sistema de antivírus e
>>  acredita-se estar livre de perigo.
>>
>>
>> =
>> Instru�ões para entrar na lista, sair da lista e usar a lista em
>> http://www.mat.puc-rio.br/~obmlistas/obm-l.html
>> =
>>
>
> --
> Esta mensagem foi verificada pelo sistema de antivírus e
> acredita-se estar livre de perigo.

-- 
Esta mensagem foi verificada pelo sistema de antiv�rus e
 acredita-se estar livre de perigo.



[obm-l] Re: [obm-l] Re: [obm-l] Re: [obm-l] Re: [obm-l] Questão OBM - U

2020-01-23 Por tôpico gilberto azevedo
Pensei em minimizar √(a² + (4-a)²)
4 - a, devido ao fato do perímetro ser 8.
No caso obtenho o mínimo sendo 2√2, quando o retângulo é um quadrado de
lado 2.
A soma das diagonais seria no caso 4√2, e não bate com o gabarito.

Em qui, 23 de jan de 2020 08:20, Bernardo Freitas Paulo da Costa <
bernardo...@gmail.com> escreveu:

> On Thu, Jan 23, 2020 at 7:24 AM gilberto azevedo 
> wrote:
> >> On Sat, Jan 11, 2020 at 11:24 AM gilberto azevedo 
> wrote:
> >> >
> >> > Qual o ínfimo sobre todos os quadriláteros convexos com perímetro
> 8 da soma dos comprimentos de suas diagonais ?
> >
> > Tentei com o retângulo e o quadrado, porém não obtive a resposta...  O
> gabarito é 4.
>
> Qual (ou quais?) retângulo(s) você testou??  Que resposta você obteve?
> --
> Bernardo Freitas Paulo da Costa
>
> --
> Esta mensagem foi verificada pelo sistema de antivírus e
>  acredita-se estar livre de perigo.
>
>
> =
> Instru�ões para entrar na lista, sair da lista e usar a lista em
> http://www.mat.puc-rio.br/~obmlistas/obm-l.html
> =
>

-- 
Esta mensagem foi verificada pelo sistema de antiv�rus e
 acredita-se estar livre de perigo.



[obm-l] Re: [obm-l] Re: [obm-l] Re: [obm-l] Questão OBM - U

2020-01-23 Por tôpico Bernardo Freitas Paulo da Costa
On Thu, Jan 23, 2020 at 7:24 AM gilberto azevedo  wrote:
>> On Sat, Jan 11, 2020 at 11:24 AM gilberto azevedo  
>> wrote:
>> >
>> > Qual o ínfimo sobre todos os quadriláteros convexos com perímetro 8 da 
>> > soma dos comprimentos de suas diagonais ?
>
> Tentei com o retângulo e o quadrado, porém não obtive a resposta...  O 
> gabarito é 4.

Qual (ou quais?) retângulo(s) você testou??  Que resposta você obteve?
-- 
Bernardo Freitas Paulo da Costa

-- 
Esta mensagem foi verificada pelo sistema de antiv�rus e
 acredita-se estar livre de perigo.


=
Instru��es para entrar na lista, sair da lista e usar a lista em
http://www.mat.puc-rio.br/~obmlistas/obm-l.html
=


[obm-l] Re: [obm-l] Re: [obm-l] Questão OBM - U

2020-01-23 Por tôpico gilberto azevedo
Tentei com o retângulo e o quadrado, porém não obtive a resposta...  O
gabarito é 4.

Em sáb, 11 de jan de 2020 12:03, Bernardo Freitas Paulo da Costa <
bernardo...@gmail.com> escreveu:

> On Sat, Jan 11, 2020 at 11:24 AM gilberto azevedo 
> wrote:
> >
> > Qual o ínfimo sobre todos os quadriláteros convexos com perímetro 8
> da soma dos comprimentos de suas diagonais ?
>
> Quais são os quadriláteros que você tentaria?
> --
> Bernardo Freitas Paulo da Costa
>
> --
> Esta mensagem foi verificada pelo sistema de antivírus e
>  acredita-se estar livre de perigo.
>
>
> =
> Instruções para entrar na lista, sair da lista e usar a lista em
> http://www.mat.puc-rio.br/~obmlistas/obm-l.html
> =
>

-- 
Esta mensagem foi verificada pelo sistema de antiv�rus e
 acredita-se estar livre de perigo.



[obm-l] Re: [obm-l] Questão OBM - U

2020-01-11 Por tôpico Bernardo Freitas Paulo da Costa
On Sat, Jan 11, 2020 at 11:24 AM gilberto azevedo  wrote:
>
> Qual o ínfimo sobre todos os quadriláteros convexos com perímetro 8 da soma 
> dos comprimentos de suas diagonais ?

Quais são os quadriláteros que você tentaria?
-- 
Bernardo Freitas Paulo da Costa

-- 
Esta mensagem foi verificada pelo sistema de antiv�rus e
 acredita-se estar livre de perigo.


=
Instru��es para entrar na lista, sair da lista e usar a lista em
http://www.mat.puc-rio.br/~obmlistas/obm-l.html
=


[obm-l] Questão OBM - U

2020-01-11 Por tôpico gilberto azevedo
Qual o ínfimo sobre todos os quadriláteros convexos com perímetro 8 da soma
dos comprimentos de suas diagonais ?

-- 
Esta mensagem foi verificada pelo sistema de antiv�rus e
 acredita-se estar livre de perigo.



[obm-l] Re: [obm-l] Questão sobre equações funcionais

2019-07-28 Por tôpico Esdras Muniz
Errei, satisfaz sim :)

Em dom, 28 de jul de 2019 14:21, Esdras Muniz 
escreveu:

> Mas essa função que VC achou não satisfaz a igualdade.
>
> Em dom, 28 de jul de 2019 01:05, Carlos Monteiro <
> cacacarlosalberto1...@gmail.com> escreveu:
>
>> (Questão) Encontre todas as funções f : R-> R tais que
>>  f(xy - f(x)) = x.f(y)
>>
>> Minha tentativa, não sei se está correta:
>> I) p(x, f(x)/(x-1)): f( f(x)/(x-1) ) = 0; x diferente de 1
>>
>> II) Seja c um número real tal que f(c)=0
>>   i) fazendo x=c na equação encontrada em I: *c diferente de 1*
>> f(0)=0
>>
>>  Se f(0)=0
>> p(x, f(x)/x): 0=x.f( f(x)/x ); x diferente de 0 .: f(x)/x=0
>>  f(x)=0, para todo x real.
>>
>>ii) p(c,1) na equação dada no problema:
>>   0=c.f(1) <=> c=0(caso anterior) ou f(1)=0
>>
>>   Se f(1)=0, 1 é raíz única
>>f( f(x)/(x-1) )=0 .: f(x)/(x-1) = 1 => f(x)=x-1, para todo x real.
>>
>>
>>
>>
>> --
>> Esta mensagem foi verificada pelo sistema de antivírus e
>> acredita-se estar livre de perigo.
>
>

-- 
Esta mensagem foi verificada pelo sistema de antiv�rus e
 acredita-se estar livre de perigo.



[obm-l] Re: [obm-l] Questão sobre equações funcionais

2019-07-28 Por tôpico Esdras Muniz
Mas essa função que VC achou não satisfaz a igualdade.

Em dom, 28 de jul de 2019 01:05, Carlos Monteiro <
cacacarlosalberto1...@gmail.com> escreveu:

> (Questão) Encontre todas as funções f : R-> R tais que
>  f(xy - f(x)) = x.f(y)
>
> Minha tentativa, não sei se está correta:
> I) p(x, f(x)/(x-1)): f( f(x)/(x-1) ) = 0; x diferente de 1
>
> II) Seja c um número real tal que f(c)=0
>   i) fazendo x=c na equação encontrada em I: *c diferente de 1*
> f(0)=0
>
>  Se f(0)=0
> p(x, f(x)/x): 0=x.f( f(x)/x ); x diferente de 0 .: f(x)/x=0
>  f(x)=0, para todo x real.
>
>ii) p(c,1) na equação dada no problema:
>   0=c.f(1) <=> c=0(caso anterior) ou f(1)=0
>
>   Se f(1)=0, 1 é raíz única
>f( f(x)/(x-1) )=0 .: f(x)/(x-1) = 1 => f(x)=x-1, para todo x real.
>
>
>
>
> --
> Esta mensagem foi verificada pelo sistema de antivírus e
> acredita-se estar livre de perigo.

-- 
Esta mensagem foi verificada pelo sistema de antiv�rus e
 acredita-se estar livre de perigo.



[obm-l] Questão sobre equações funcionais

2019-07-27 Por tôpico Carlos Monteiro
(Questão) Encontre todas as funções f : R-> R tais que
 f(xy - f(x)) = x.f(y)

Minha tentativa, não sei se está correta:
I) p(x, f(x)/(x-1)): f( f(x)/(x-1) ) = 0; x diferente de 1

II) Seja c um número real tal que f(c)=0
  i) fazendo x=c na equação encontrada em I: *c diferente de 1*
f(0)=0

 Se f(0)=0
p(x, f(x)/x): 0=x.f( f(x)/x ); x diferente de 0 .: f(x)/x=0
 f(x)=0, para todo x real.

   ii) p(c,1) na equação dada no problema:
  0=c.f(1) <=> c=0(caso anterior) ou f(1)=0

  Se f(1)=0, 1 é raíz única
   f( f(x)/(x-1) )=0 .: f(x)/(x-1) = 1 => f(x)=x-1, para todo x real.

-- 
Esta mensagem foi verificada pelo sistema de antiv�rus e
 acredita-se estar livre de perigo.



[obm-l] Re: [obm-l] OBM 2018 - Nível 1 - Questão 2

2019-03-26 Por tôpico Pedro José
Boa noite!

Não mencionei que embora no braço não seja um trabalho hercúleo, pois, o
resto por 9, se obtém com somas consecutivas dos algarismos.

1, 2, 2, 4, 8, 5 (3+2), 4 (4+0), 2 ,8, 7, 2, 5, 1, 5, 5, 7, 8, 2, 7, 5, 8,
4, 5, 2, 1, 2... Pronto achado o período 24.

De toda sorte deve ter forma mais fácil.

Saudações,
PJMS


Em ter, 26 de mar de 2019 às 19:11, Pedro José 
escreveu:

> Boa noite!
>
> a) O quarto termo é nulo e a partir daí todos também são.
> b) Esse, deve ter uma solução mais elegante. Fiz no braço e dá um período
> de 24. Logo dá o mesmo termo da ordem do resto de 2018 por 24 que é 2.
> Portanto, dá o segundo termo que por coincidência é 2.
> c) 1,1,1,1,1,1,1,1,1,1,1,1,1,1,1,1 e
> 1,10,1,1,1,1,1,1,1,1,1,1,1,1,1.
>
> Quanto ao b deve ter um modo de encontrar o período, sem ser no braço.
>
> Saudações,
> PJMS.
>
> Em ter, 26 de mar de 2019 às 18:24,  escreveu:
>
>> 2. Considere abaixo as sequências de números inteiros que possuem as duas
>> propriedades a seguir:
>>
>> i) Os dois primeiros termos são dados.
>> ii) Cada um dos termos seguintes e o resto da divisão por 9 do produto
>> dos dois termos anteriores.
>>
>> a) Qual e o vigésimo termo da sequência: 2, 3, ...?
>> b) Qual e o 2018 o termo da sequencia 1, 2, ...?
>> c) Apresente duas sequências cujo 2018 o termo e igual a 1.
>>
>>
>>
>> --
>> Esta mensagem foi verificada pelo sistema de antivírus e
>> acredita-se estar livre de perigo.
>>
>

-- 
Esta mensagem foi verificada pelo sistema de antiv�rus e
 acredita-se estar livre de perigo.



[obm-l] Re: [obm-l] OBM 2018 - Nível 1 - Questão 2

2019-03-26 Por tôpico Pedro José
Boa noite!

a) O quarto termo é nulo e a partir daí todos também são.
b) Esse, deve ter uma solução mais elegante. Fiz no braço e dá um período
de 24. Logo dá o mesmo termo da ordem do resto de 2018 por 24 que é 2.
Portanto, dá o segundo termo que por coincidência é 2.
c) 1,1,1,1,1,1,1,1,1,1,1,1,1,1,1,1 e 1,10,1,1,1,1,1,1,1,1,1,1,1,1,1.

Quanto ao b deve ter um modo de encontrar o período, sem ser no braço.

Saudações,
PJMS.

Em ter, 26 de mar de 2019 às 18:24,  escreveu:

> 2. Considere abaixo as sequências de números inteiros que possuem as duas
> propriedades a seguir:
>
> i) Os dois primeiros termos são dados.
> ii) Cada um dos termos seguintes e o resto da divisão por 9 do produto dos
> dois termos anteriores.
>
> a) Qual e o vigésimo termo da sequência: 2, 3, ...?
> b) Qual e o 2018 o termo da sequencia 1, 2, ...?
> c) Apresente duas sequências cujo 2018 o termo e igual a 1.
>
>
>
> --
> Esta mensagem foi verificada pelo sistema de antivírus e
> acredita-se estar livre de perigo.
>

-- 
Esta mensagem foi verificada pelo sistema de antiv�rus e
 acredita-se estar livre de perigo.



[obm-l] OBM 2018 - Nível 1 - Questão 2

2019-03-26 Por tôpico jjunior
2. Considere abaixo as sequências de números inteiros que possuem as
duas propriedades a seguir: 

i) Os dois primeiros termos são dados. 
ii) Cada um dos termos seguintes e o resto da divisão por 9 do
produto dos dois termos anteriores. 

a) Qual e o vigésimo termo da sequência: 2, 3, ...? 
b) Qual e o 2018 o termo da sequencia 1, 2, ...? 
c) Apresente duas sequências cujo 2018 o termo e igual a 1.



-- 
Esta mensagem foi verificada pelo sistema de antiv�rus e
 acredita-se estar livre de perigo.



[obm-l] Re: [obm-l] Re: [obm-l] Questão de probabilidade

2018-11-06 Por tôpico Vanderlei Nemitz
Bela solução, Bruno!
Muito obrigado!

Em ter, 6 de nov de 2018 15:38, Bruno Visnadi  Seja Pa a probabilidade de ocorrência de a. Defina Pb e Pc analogamente.
> a = Pa(1-Pb)(1-Pc)
> b = Pb(1-Pa)(1-Pc)
> c = Pc(1-Pa)(1-Pb)
> p = (1-Pa)(1-Pb)(1-Pc)
> Queremos achar a razão Pa/Pc
> Da equação (a - 2b)p = ab, obtemos:
> (1-Pa)(1-Pb)(1-Pc)²(Pa(1-Pb) - 2Pb(1-Pa)) = PaPb(1-Pa)(1-Pb)(1-Pc)²
> Pa(1-Pb) - 2Pb(1-Pa)  = PaPb
> Pa - 2Pb + PaPb = PaPb
> Pa = 2Pb -> Pb = Pa/2
> Da equação  (b - 3c)p = 2bc, obtemos:
> (1-Pa)²(1-Pb)(1-Pc)(Pb(1-Pc) - 3Pc(1-Pb)) = 2PbPc(1-Pa)²(1-Pb)(1-Pc)
> Pb(1-Pc) - 3Pc(1-Pb) = 2PbPc
> Pb - 3Pc + 2PcPb = 2PbPc
> Pb = 3Pc
> Logo: Pa/2 = 3Pc
> Pa/Pc = 6
>
>
>
>
>
> Em ter, 6 de nov de 2018 às 12:43, Vanderlei Nemitz 
> escreveu:
>
>> Pessoal, alguém tem um ideia de como resolver a seguinte questão? Já
>> tentei muita coisa, sem sucesso.
>> Muito obrigado!
>>
>> Vanderlei
>>
>> Sejam três eventos independentes A, B e C. A probabilidade de que ocorra
>> apenas o evento A é a, apenas o evento B é b e apenas o evento C é c. Seja
>> p a probabilidade de que nenhum dos eventos A, B ou C ocorra. Sabendo que
>> todas as probabilidades citadas são números no intervalo ]0, 1[ e que p
>> satisfaz as equações (a - 2b).p = ab e (b - 3c).p = 2bc, a razão entre a
>> probabilidade de ocorrência de A e a probabilidade de ocorrência de C é:
>> a) 12
>> b) 3
>> c) 10
>> d) 5
>> e) 6
>>
>> --
>> Esta mensagem foi verificada pelo sistema de antivírus e
>> acredita-se estar livre de perigo.
>
>
> --
> Esta mensagem foi verificada pelo sistema de antivírus e
> acredita-se estar livre de perigo.

-- 
Esta mensagem foi verificada pelo sistema de antiv�rus e
 acredita-se estar livre de perigo.



[obm-l] Re: [obm-l] Questão de probabilidade

2018-11-06 Por tôpico Bruno Visnadi
Seja Pa a probabilidade de ocorrência de a. Defina Pb e Pc analogamente.
a = Pa(1-Pb)(1-Pc)
b = Pb(1-Pa)(1-Pc)
c = Pc(1-Pa)(1-Pb)
p = (1-Pa)(1-Pb)(1-Pc)
Queremos achar a razão Pa/Pc
Da equação (a - 2b)p = ab, obtemos:
(1-Pa)(1-Pb)(1-Pc)²(Pa(1-Pb) - 2Pb(1-Pa)) = PaPb(1-Pa)(1-Pb)(1-Pc)²
Pa(1-Pb) - 2Pb(1-Pa)  = PaPb
Pa - 2Pb + PaPb = PaPb
Pa = 2Pb -> Pb = Pa/2
Da equação  (b - 3c)p = 2bc, obtemos:
(1-Pa)²(1-Pb)(1-Pc)(Pb(1-Pc) - 3Pc(1-Pb)) = 2PbPc(1-Pa)²(1-Pb)(1-Pc)
Pb(1-Pc) - 3Pc(1-Pb) = 2PbPc
Pb - 3Pc + 2PcPb = 2PbPc
Pb = 3Pc
Logo: Pa/2 = 3Pc
Pa/Pc = 6





Em ter, 6 de nov de 2018 às 12:43, Vanderlei Nemitz 
escreveu:

> Pessoal, alguém tem um ideia de como resolver a seguinte questão? Já
> tentei muita coisa, sem sucesso.
> Muito obrigado!
>
> Vanderlei
>
> Sejam três eventos independentes A, B e C. A probabilidade de que ocorra
> apenas o evento A é a, apenas o evento B é b e apenas o evento C é c. Seja
> p a probabilidade de que nenhum dos eventos A, B ou C ocorra. Sabendo que
> todas as probabilidades citadas são números no intervalo ]0, 1[ e que p
> satisfaz as equações (a - 2b).p = ab e (b - 3c).p = 2bc, a razão entre a
> probabilidade de ocorrência de A e a probabilidade de ocorrência de C é:
> a) 12
> b) 3
> c) 10
> d) 5
> e) 6
>
> --
> Esta mensagem foi verificada pelo sistema de antivírus e
> acredita-se estar livre de perigo.

-- 
Esta mensagem foi verificada pelo sistema de antiv�rus e
 acredita-se estar livre de perigo.



[obm-l] Questão de probabilidade

2018-11-06 Por tôpico Vanderlei Nemitz
Pessoal, alguém tem um ideia de como resolver a seguinte questão? Já tentei
muita coisa, sem sucesso.
Muito obrigado!

Vanderlei

Sejam três eventos independentes A, B e C. A probabilidade de que ocorra
apenas o evento A é a, apenas o evento B é b e apenas o evento C é c. Seja
p a probabilidade de que nenhum dos eventos A, B ou C ocorra. Sabendo que
todas as probabilidades citadas são números no intervalo ]0, 1[ e que p
satisfaz as equações (a - 2b).p = ab e (b - 3c).p = 2bc, a razão entre a
probabilidade de ocorrência de A e a probabilidade de ocorrência de C é:
a) 12
b) 3
c) 10
d) 5
e) 6

-- 
Esta mensagem foi verificada pelo sistema de antiv�rus e
 acredita-se estar livre de perigo.



[obm-l] Re: [obm-l] Re: [obm-l] Questão do ITA

2018-10-11 Por tôpico Vanderlei Nemitz
Valeu, Ralph!
Como sempre, uma explicação clara e simples!

Em qua, 10 de out de 2018 17:05, Ralph Teixeira 
escreveu:

> Note que x=5 é um possível valor que resolve aquela equação (mas,
> sinceramente, não interessa, eu faria o raciocínio abaixo com qualquer
> número).
>
> Então qualquer polinômio que satisfaça f(1)=5, f(-1)=10 e f(0)=20
> automaticamente satisfaz todas as condições do enunciado (note que
> a_0=f(0)). Em outras palavras, qualquer polinômio cujo gráfico passe pelos
> pontos (-1,10),(0,20),(1,5) serve.
>
> Agora escolha um ponto (z,0) qualquer como 4o ponto (onde z não é -1, 0
> nem 1). Como quaisquer 4 pontos (com "x"s diferentes) determinam um único
> polinômio de grau 3, haverá um polinômio de grau 3 que passa pelos pontos
> dados e que tem raiz z. Como z pode ser negativo, positivo, raiz(2), ou 42,
> nenhuma das respostas (A)-(D) pode valer (respectivamente!). Então tem que
> ser (E).
>
> Abraço, Ralph.
>
> On Wed, Oct 10, 2018 at 5:41 AM Vanderlei Nemitz 
> wrote:
>
>> Bom dia, pessoal!
>> Encontrei essa questão, que diz ser do ITA (eu particularmente não
>> encontrei na internet).
>> Como a resposta é E (nenhuma das anteriores), não sei se é possível
>> provar que as anteriores são falsas. Eu não consegui concluir coisa alguma.
>>
>> *Seja f(x) = am.x^m + am–1.x^(m–1) + ... + a1.x + a0, onde am, am–1, ...,
>> a1, a0 são reais, am diferente de 0 e a0 diferente de 0. Se f(1) é solução
>> real da equação 2^(x–3) + 2^(x–4) = 2^(x–2) – 2^(x–1) + 14, f(–1) = 2.f(1)
>> e a0 = 2.f(–1), então podemos afirmar:*
>>
>> *a) f(x) tem somente raízes reais positivas.*
>>
>> *b) f(x) tem somente raízes reais negativas.*
>>
>> *c) f(x) tem somente raízes reais inteiras.*
>>
>> *d) f(x) não tem raízes reais inteiras.*
>>
>> *e) nda*
>> Alguém tem alguma ideia?
>> Muito obrigado!
>>
>> --
>> Esta mensagem foi verificada pelo sistema de antivírus e
>> acredita-se estar livre de perigo.
>
>
> --
> Esta mensagem foi verificada pelo sistema de antivírus e
> acredita-se estar livre de perigo.

-- 
Esta mensagem foi verificada pelo sistema de antiv�rus e
 acredita-se estar livre de perigo.



[obm-l] Re: [obm-l] Questão do ITA

2018-10-10 Por tôpico Ralph Teixeira
Note que x=5 é um possível valor que resolve aquela equação (mas,
sinceramente, não interessa, eu faria o raciocínio abaixo com qualquer
número).

Então qualquer polinômio que satisfaça f(1)=5, f(-1)=10 e f(0)=20
automaticamente satisfaz todas as condições do enunciado (note que
a_0=f(0)). Em outras palavras, qualquer polinômio cujo gráfico passe pelos
pontos (-1,10),(0,20),(1,5) serve.

Agora escolha um ponto (z,0) qualquer como 4o ponto (onde z não é -1, 0 nem
1). Como quaisquer 4 pontos (com "x"s diferentes) determinam um único
polinômio de grau 3, haverá um polinômio de grau 3 que passa pelos pontos
dados e que tem raiz z. Como z pode ser negativo, positivo, raiz(2), ou 42,
nenhuma das respostas (A)-(D) pode valer (respectivamente!). Então tem que
ser (E).

Abraço, Ralph.

On Wed, Oct 10, 2018 at 5:41 AM Vanderlei Nemitz 
wrote:

> Bom dia, pessoal!
> Encontrei essa questão, que diz ser do ITA (eu particularmente não
> encontrei na internet).
> Como a resposta é E (nenhuma das anteriores), não sei se é possível provar
> que as anteriores são falsas. Eu não consegui concluir coisa alguma.
>
> *Seja f(x) = am.x^m + am–1.x^(m–1) + ... + a1.x + a0, onde am, am–1, ...,
> a1, a0 são reais, am diferente de 0 e a0 diferente de 0. Se f(1) é solução
> real da equação 2^(x–3) + 2^(x–4) = 2^(x–2) – 2^(x–1) + 14, f(–1) = 2.f(1)
> e a0 = 2.f(–1), então podemos afirmar:*
>
> *a) f(x) tem somente raízes reais positivas.*
>
> *b) f(x) tem somente raízes reais negativas.*
>
> *c) f(x) tem somente raízes reais inteiras.*
>
> *d) f(x) não tem raízes reais inteiras.*
>
> *e) nda*
> Alguém tem alguma ideia?
> Muito obrigado!
>
> --
> Esta mensagem foi verificada pelo sistema de antivírus e
> acredita-se estar livre de perigo.

-- 
Esta mensagem foi verificada pelo sistema de antiv�rus e
 acredita-se estar livre de perigo.



[obm-l] Questão do ITA

2018-10-10 Por tôpico Vanderlei Nemitz
Bom dia, pessoal!
Encontrei essa questão, que diz ser do ITA (eu particularmente não
encontrei na internet).
Como a resposta é E (nenhuma das anteriores), não sei se é possível provar
que as anteriores são falsas. Eu não consegui concluir coisa alguma.

*Seja f(x) = am.x^m + am–1.x^(m–1) + ... + a1.x + a0, onde am, am–1, ...,
a1, a0 são reais, am diferente de 0 e a0 diferente de 0. Se f(1) é solução
real da equação 2^(x–3) + 2^(x–4) = 2^(x–2) – 2^(x–1) + 14, f(–1) = 2.f(1)
e a0 = 2.f(–1), então podemos afirmar:*

*a) f(x) tem somente raízes reais positivas.*

*b) f(x) tem somente raízes reais negativas.*

*c) f(x) tem somente raízes reais inteiras.*

*d) f(x) não tem raízes reais inteiras.*

*e) nda*
Alguém tem alguma ideia?
Muito obrigado!

-- 
Esta mensagem foi verificada pelo sistema de antiv�rus e
 acredita-se estar livre de perigo.



[obm-l] Re: [obm-l] Questão do Gandhi

2018-08-15 Por tôpico gilberto azevedo
Só uma ressalva, alí depois do "ou a+1 será par, e a ... "
Não tem esse "a" no final, erro de digitação.

Em Qua, 15 de ago de 2018 18:02, gilberto azevedo 
escreveu:

> Supondo que b>a, então b = a+1
> Logo :
> D = a² + (a+1)² + (a*(a+1))²
> D = a² + a² + 2a + 1 + (a²+a)²
> D = 2a² + 2a + 1 + (a²+a)²
> D = 2(a²+a) + 1 + (a²+a)²
> D = (a²+a)² + 2(a²+a) + 1 (só organizei)
> Agora a sacada é perceber que está na forma x²+2xy+y² sendo x = a²+a e y =
> 1
> Logo :
> D = (a²+a+1)²
> √D = a²+a+1
> √D = a(a+1) + 1
> Agora basta analisar que :
> a(a+1) é sempre par pois ou a ou a+1 será par, e a somando com 1 irá
> formar um número ímpar.
> Assim a raíz é inteira e é sempre ímpar !
> Espero ter ajudado. Abs.
>
>
> Em 15 de ago de 2018 17:30, "Daniel Quevedo" 
> escreveu:
>
> Seja D = a^2 + b^2 + c^2, onde a e b são inteiros consecutivos e c = a•b.
> Então sobre a raiz quadrada de D podemos afirmar que:
>
> A) é sempre inteiro par
> B) algumas vezes é inteiro par, outras vezes não.
> C) algumas vezes é racional, outras vezes não.
> D) é sempre inteiro ímpar.
> E) é sempre irracional.
>
> Gab: d
>
> PS: é fácil mostrar q D é inteiro ímpar, minha dificuldade está em mostrar
> q a raiz quadrada tbm é.
> --
> Fiscal: Daniel Quevedo
>
> --
> Esta mensagem foi verificada pelo sistema de antivírus e
> acredita-se estar livre de perigo.
>
>
>

-- 
Esta mensagem foi verificada pelo sistema de antiv�rus e
 acredita-se estar livre de perigo.



[obm-l] Re: [obm-l] Questão do Gandhi

2018-08-15 Por tôpico Claudio Buffara
D = a^2 + (a+1)^2 + a^2*(a+1)^2 = a^4 + 2a^3 + 3a^2 + 2a + 1.

Se D for um quadrado, então será da forma (a^2 + a + x)^2.

Expandindo isso e comparando coeficientes, obtemos x = 1 ==> D = (a^2 + a +
1)^2.

Como a^2 + a é par, raiz(D) = a^2 + a + 1 é ímpar.

[]s,
Claudio.


2018-08-15 17:22 GMT-03:00 Daniel Quevedo :

> Seja D = a^2 + b^2 + c^2, onde a e b são inteiros consecutivos e c = a•b.
> Então sobre a raiz quadrada de D podemos afirmar que:
>
> A) é sempre inteiro par
> B) algumas vezes é inteiro par, outras vezes não.
> C) algumas vezes é racional, outras vezes não.
> D) é sempre inteiro ímpar.
> E) é sempre irracional.
>
> Gab: d
>
> PS: é fácil mostrar q D é inteiro ímpar, minha dificuldade está em mostrar
> q a raiz quadrada tbm é.
> --
> Fiscal: Daniel Quevedo
>
> --
> Esta mensagem foi verificada pelo sistema de antivírus e
> acredita-se estar livre de perigo.

-- 
Esta mensagem foi verificada pelo sistema de antiv�rus e
 acredita-se estar livre de perigo.



[obm-l] Re: [obm-l] Questão do Gandhi

2018-08-15 Por tôpico gilberto azevedo
Supondo que b>a, então b = a+1
Logo :
D = a² + (a+1)² + (a*(a+1))²
D = a² + a² + 2a + 1 + (a²+a)²
D = 2a² + 2a + 1 + (a²+a)²
D = 2(a²+a) + 1 + (a²+a)²
D = (a²+a)² + 2(a²+a) + 1 (só organizei)
Agora a sacada é perceber que está na forma x²+2xy+y² sendo x = a²+a e y = 1
Logo :
D = (a²+a+1)²
√D = a²+a+1
√D = a(a+1) + 1
Agora basta analisar que :
a(a+1) é sempre par pois ou a ou a+1 será par, e a somando com 1 irá formar
um número ímpar.
Assim a raíz é inteira e é sempre ímpar !
Espero ter ajudado. Abs.


Em 15 de ago de 2018 17:30, "Daniel Quevedo"  escreveu:

Seja D = a^2 + b^2 + c^2, onde a e b são inteiros consecutivos e c = a•b.
Então sobre a raiz quadrada de D podemos afirmar que:

A) é sempre inteiro par
B) algumas vezes é inteiro par, outras vezes não.
C) algumas vezes é racional, outras vezes não.
D) é sempre inteiro ímpar.
E) é sempre irracional.

Gab: d

PS: é fácil mostrar q D é inteiro ímpar, minha dificuldade está em mostrar
q a raiz quadrada tbm é.
-- 
Fiscal: Daniel Quevedo

-- 
Esta mensagem foi verificada pelo sistema de antivírus e
acredita-se estar livre de perigo.

-- 
Esta mensagem foi verificada pelo sistema de antiv�rus e
 acredita-se estar livre de perigo.



[obm-l] Questão do Gandhi

2018-08-15 Por tôpico Daniel Quevedo
Seja D = a^2 + b^2 + c^2, onde a e b são inteiros consecutivos e c = a•b.
Então sobre a raiz quadrada de D podemos afirmar que:

A) é sempre inteiro par
B) algumas vezes é inteiro par, outras vezes não.
C) algumas vezes é racional, outras vezes não.
D) é sempre inteiro ímpar.
E) é sempre irracional.

Gab: d

PS: é fácil mostrar q D é inteiro ímpar, minha dificuldade está em mostrar
q a raiz quadrada tbm é.
-- 
Fiscal: Daniel Quevedo

-- 
Esta mensagem foi verificada pelo sistema de antiv�rus e
 acredita-se estar livre de perigo.



[obm-l] Re: [obm-l] Re: [obm-l] Questão do IME

2018-07-14 Por tôpico Vanderlei Nemitz
Muito obrigado, Claudio!
Bela solução!

Em 13 de julho de 2018 13:35, Claudio Buffara 
escreveu:

> Os prolongamentos de DM e EN se intersectam num mesmo ponto P pertencente
> a AB.
> Pra ver isso, repare que os triângulos DCM e PAM são semelhantes (razão de
> semelhança = 2).
> Idem para os triângulos EFN e PNB.
> Como, no triângulo PDE (que é isósceles), vale PM/PD = PN/PE = 1/3,
> concluímos que MN é paralelo a DE.
>
> []s,
> Claudio.
>
>
> 2018-07-13 12:13 GMT-03:00 Vanderlei Nemitz :
>
>> Sejam dois quadrados ABCD e ABEF, tendo um lado comum AB, mas não
>> situados num mesmo plano. Sejam M e N pertencentes, respectivamente, às
>> diagonais AC e BF tais que AM/AC = BN/BF = 1/3. Mostre que MN é paralelo a
>> DE.
>>
>> Alguém poderia ajudar?
>> Obrigado,
>> Vanderlei
>>
>> --
>> Esta mensagem foi verificada pelo sistema de antivírus e
>> acredita-se estar livre de perigo.
>
>
>
> --
> Esta mensagem foi verificada pelo sistema de antivírus e
> acredita-se estar livre de perigo.

-- 
Esta mensagem foi verificada pelo sistema de antiv�rus e
 acredita-se estar livre de perigo.



Re: [obm-l] Re: [obm-l] Questão do IME

2018-07-14 Por tôpico wagner

Brilhante!


Quoting Claudio Buffara :


Os prolongamentos de DM e EN se intersectam num mesmo ponto P pertencente a
AB.
Pra ver isso, repare que os triângulos DCM e PAM são semelhantes (razão de
semelhança = 2).
Idem para os triângulos EFN e PNB.
Como, no triângulo PDE (que é isósceles), vale PM/PD = PN/PE = 1/3,
concluímos que MN é paralelo a DE.

[]s,
Claudio.


2018-07-13 12:13 GMT-03:00 Vanderlei Nemitz :


Sejam dois quadrados ABCD e ABEF, tendo um lado comum AB, mas não situados
num mesmo plano. Sejam M e N pertencentes, respectivamente, às diagonais AC
e BF tais que AM/AC = BN/BF = 1/3. Mostre que MN é paralelo a DE.

Alguém poderia ajudar?
Obrigado,
Vanderlei

--
Esta mensagem foi verificada pelo sistema de antivírus e
acredita-se estar livre de perigo.


--
Esta mensagem foi verificada pelo sistema de antiv?rus e
 acredita-se estar livre de perigo.





--
Esta mensagem foi verificada pelo sistema de antiv�rus e
acredita-se estar livre de perigo.

=
Instru��es para entrar na lista, sair da lista e usar a lista em
http://www.mat.puc-rio.br/~obmlistas/obm-l.html
=


Re: [obm-l] Questão do IME

2018-07-13 Por tôpico Ary Medino
Seguem algumas ideias para uma solução por vetores em R^3, mas precisa esboçar 
as figuras para ficar mais claro.
- Sem perda de generalidade, assuma que os quadrados têm lado medindo 1.

- Esboce a figura no R^3, sendo B a origem do sistema, o lado AB no eixo x e o 
lado BC no eixo y.

- Chame de t o ângulo no plano yz entre os lados BC e BE.
Então temos o vetor MN = (-1, cos(t)-1, sen(t))  e o vetor DE = (1/3) (-1, 
cos(t)-1, sen(t)), ou seja esses vetores satisfazem MN = 3DE e em particular 
eles são paralelos. 
Obs. Pense como se fosse um caderno quadrado cuja lina da espiral é o lado AB, 
a capa ABCD está apoiada no plano horizontal xy e a oura capa ABEF vc pode 
abrir e fechar como quiser, formando um ângulo t entre elas.

AbraçoAry 

Em Sexta-feira, 13 de Julho de 2018 13:11, Vanderlei Nemitz 
 escreveu:
 

 Sejam dois quadrados ABCD e ABEF, tendo um lado comum AB, mas não situados num 
mesmo plano. Sejam M e N pertencentes, respectivamente, às diagonais AC e BF 
tais que AM/AC = BN/BF = 1/3. Mostre que MN é paralelo a DE.
Alguém poderia ajudar?Obrigado,Vanderlei
--
Esta mensagem foi verificada pelo sistema de antiv�rus e 
 acredita-se estar livre de perigo.

   
 
|  | Livre de vírus. www.avast.com.  |


-- 
Esta mensagem foi verificada pelo sistema de antiv�rus e
 acredita-se estar livre de perigo.



[obm-l] Re: [obm-l] Questão do IME

2018-07-13 Por tôpico Claudio Buffara
Os prolongamentos de DM e EN se intersectam num mesmo ponto P pertencente a
AB.
Pra ver isso, repare que os triângulos DCM e PAM são semelhantes (razão de
semelhança = 2).
Idem para os triângulos EFN e PNB.
Como, no triângulo PDE (que é isósceles), vale PM/PD = PN/PE = 1/3,
concluímos que MN é paralelo a DE.

[]s,
Claudio.


2018-07-13 12:13 GMT-03:00 Vanderlei Nemitz :

> Sejam dois quadrados ABCD e ABEF, tendo um lado comum AB, mas não situados
> num mesmo plano. Sejam M e N pertencentes, respectivamente, às diagonais AC
> e BF tais que AM/AC = BN/BF = 1/3. Mostre que MN é paralelo a DE.
>
> Alguém poderia ajudar?
> Obrigado,
> Vanderlei
>
> --
> Esta mensagem foi verificada pelo sistema de antivírus e
> acredita-se estar livre de perigo.

-- 
Esta mensagem foi verificada pelo sistema de antiv�rus e
 acredita-se estar livre de perigo.



[obm-l] Questão do IME

2018-07-13 Por tôpico Vanderlei Nemitz
Sejam dois quadrados ABCD e ABEF, tendo um lado comum AB, mas não situados
num mesmo plano. Sejam M e N pertencentes, respectivamente, às diagonais AC
e BF tais que AM/AC = BN/BF = 1/3. Mostre que MN é paralelo a DE.

Alguém poderia ajudar?
Obrigado,
Vanderlei

-- 
Esta mensagem foi verificada pelo sistema de antiv�rus e
 acredita-se estar livre de perigo.



[obm-l] Questão da XXII olimpiada de mayo

2018-06-15 Por tôpico Pedro José
Bom dia!

Alguém poderia postar uma solução de um problema da XXII olimpiada, mais
especificamente o item b) :Dizemos que um número inteiro positivo é
qua-divi se é divisível pela soma dos quadrados de seus dígitos, e além
disso nenhum de seus dígitos é igual a zero.
a) Encontre um número qua-divi tal que a soma de seus dígitos seja 24.
b) Encontre um número qua-divi tal que a soma de seus dígitos seja 1001.

Creio ter conseguido, mas foi de orelhada. Gostaria de ver uma solução mais
balizada.
Criei um número com fatores congruentes a 1 mod 6, exceto o 5 e o11.
Além disso a ordem de 10 mod desses fatores é sempre 6, exceto o 5(que não
tem ordem 10 mod5) e o 11 que será 2, melhor. Mas o 5 não tem problema.
Então o objetivo é formar um número da seguinte forma:
...AB...B...C concatenado com o número criado, mencionado
anteriormente.
O número criado foi:
84.259.175 = 5^2*7^2*11*13^2*37
Então a soma dos algarismos desse número é 41 e dos quadrados de seus
algarismos é 265.
No número que pretendo formar o número de algarismos em bloco será múltiplo
de 6.
Então fica o sistema para apenas dois blocos:
ax+by= (1001-41)/6=160
a^2*x +b^2*y=(S2 -265)/6.
Onde x e y é a quantidade de repetições de blocos de 6 algarismos e a e b
são os algarismos e S2 é a soma dos quadrados de todos dígitos.
Agora preciso criar S2 que feche com o problema. Tem que ser 1 mod 6, para
quando subtrair 265, ser divisível por 6. Deve ser um divisor do número
criado no início. 5^2*7^2*11*13^2*37.
Seja S2=5005=5*7*11*13
xa+yb=160
xa^2+yb^2= (5005-265)/6=790.
Como 6| 790 - 160, 1 e 3 formam uma boa escolha, mas infortunadamente, o
número de blocos de 1 dá negativo.
Então introduzi 9 blocos de 8 para acertar, já que há liberdade.

Aí dão 9 blocos de 8, 25 blocos de 1 e 21 blocos de três, concatenado ao
final com 84.259.175.
É o número fica.
10^274*8*(10^54-1)/9+10^124*(10^150-1)/9+10^8*3*(10^126)/9+
5^2*7^2*11*13^2*17.
Como
10^6 =1 mod p, com p=7 ou p=11 ou p= 13 e 5 |10, S2=5*7*11*13, S2 divide
cada parcela e portanto o número.
O número são 54 algarismos 8, seguidos de 150 algarismos 1,seguidos de126
algarismos 3 seguidos de 84259175.
Sem ser o 84259175 que tem que ser assim ao final, ainda atendem todas as
permutações dos blocos de 6: 67! /(9!.25!.23!)

Saudações,
PJMS

-- 
Esta mensagem foi verificada pelo sistema de antiv�rus e
 acredita-se estar livre de perigo.



[obm-l] Re: [obm-l] Questão sobre divisor primo

2018-06-06 Por tôpico Otávio Araújo
Só uma curiosidade: de onde é essa questão?

Em qua, 6 de jun de 2018 11:38, Pedro Chaves 
escreveu:

> Caros Colegas,
>
> Não consegui resolver a questão abaixo. Peço auxílio.
>
> Questão:Para cada inteiro positivo n, mostrar que todo divisor primo
> de 12n^2 + 1 é da forma 6k +1, sendo k um inteiro positivo.
>
>
>
>
> <https://www.avast.com/sig-email?utm_medium=email_source=link_campaign=sig-email_content=webmail>
>  Livre
> de vírus. www.avast.com
> <https://www.avast.com/sig-email?utm_medium=email_source=link_campaign=sig-email_content=webmail>.
> <#m_-594041030565945_DAB4FAD8-2DD7-40BB-A1B8-4E2AA1F9FDF2>
>
> --
> Esta mensagem foi verificada pelo sistema de antivírus e
> acredita-se estar livre de perigo.
>

-- 
Esta mensagem foi verificada pelo sistema de antiv�rus e
 acredita-se estar livre de perigo.



[obm-l] Re: [obm-l] Questão sobre divisor primo

2018-06-06 Por tôpico Otávio Araújo
Só um detalhe que errei na digitação:
1 = (1/p) = (x.12/p) = ((-x).(-12)/p)=
(-x/p)(-12/p)

Em qua, 6 de jun de 2018 15:55, Otávio Araújo 
escreveu:

> Tenho uma solução aqui:
> Seja p um primo que divide 12n^2 +1, teremos que 12n^2 = -1 mód p. Seja x
> o inverso multiplicativo de 12 módulo p em (Zp)*, então n^2= -x mód p,
> portanto
> -x é resíduo quadrático módulo p. Denote (/) o simbolo de Legendre,
> teremos (-x/p)=1, mas 1=(1/p)=(x.12/p)
> (-x/p)(-12/p) --> (-12/p)=1 -->
> (-1/p)(4/p)(3/p)=1 --> (-1/p)(3/p)=1, ou seja, (-1/p)=(3/p)=1 ou
> (-1/p)=(3/p)=-1.
> Em qualquer livro de teoria dos números que contenha o assunto de resíduos
> quadráticos podemos ver a demonstração de que
> (-1/p)=1 se p=1 mód 4 e (-1/p)=-1 se
>  p=3 mód 4.
> Do teorema da reciprocidade quadrática (p é diferente de 2 e 3), temos
> (p/3)(3/p)= (-1)^((p-1)/2) -->
> Mas p é congruente a 1,5 ,7 ou 11 módulo 12 ( pois é primo e não é 2 ou
> 3), testando cada caso, temos (p/3)=1 se p=1 ou 7 módulo 12 (pois devemos
> ter p=1 mód 3) e (p/3)=-1 se p=5 ou 11 módulo 12.  Observando que
> (-1)^((p-1)/2)=1 se p=1 ou 5 módulo 12 e -1 se p=7 ou 11 módulo 12,
> obteremos que (3/p)=1 se p =1 ou 11 módulo 12 e -1 se p =5 ou 7 módulo 12.
>
> Por último, se (-1/p)=(3/p)=1, teremos
> p=1 mód 4 e p= 1 ou 11 módulo 12
> --> p=1 mód 12.
> Se (-1/p)=(3/p)=-1, teremos
> p=3 mód 4 e p=5 ou 7 módulo 12
> --> p=7 mód 12.
> Daí p = 1 ou 7 módulo 12 --> p=1 mód 6, como queríamos.
>
> Em qua, 6 de jun de 2018 11:38, Pedro Chaves 
> escreveu:
>
>> Caros Colegas,
>>
>> Não consegui resolver a questão abaixo. Peço auxílio.
>>
>> Questão:Para cada inteiro positivo n, mostrar que todo divisor primo
>> de 12n^2 + 1 é da forma 6k +1, sendo k um inteiro positivo.
>>
>>
>>
>>
>> <https://www.avast.com/sig-email?utm_medium=email_source=link_campaign=sig-email_content=webmail>
>>  Livre
>> de vírus. www.avast.com
>> <https://www.avast.com/sig-email?utm_medium=email_source=link_campaign=sig-email_content=webmail>.
>>
>> <#m_3212190965219518123_m_8119351896224020777_m_-594041030565945_DAB4FAD8-2DD7-40BB-A1B8-4E2AA1F9FDF2>
>>
>> --
>> Esta mensagem foi verificada pelo sistema de antivírus e
>> acredita-se estar livre de perigo.
>>
>

-- 
Esta mensagem foi verificada pelo sistema de antiv�rus e
 acredita-se estar livre de perigo.



[obm-l] Re: [obm-l] Questão sobre divisor primo

2018-06-06 Por tôpico Otávio Araújo
Tenho uma solução aqui:
Seja p um primo que divide 12n^2 +1, teremos que 12n^2 = -1 mód p. Seja x o
inverso multiplicativo de 12 módulo p em (Zp)*, então n^2= -x mód p,
portanto
-x é resíduo quadrático módulo p. Denote (/) o simbolo de Legendre, teremos
(-x/p)=1, mas 1=(1/p)=(x.12/p)
(-x/p)(-12/p) --> (-12/p)=1 -->
(-1/p)(4/p)(3/p)=1 --> (-1/p)(3/p)=1, ou seja, (-1/p)=(3/p)=1 ou
(-1/p)=(3/p)=-1.
Em qualquer livro de teoria dos números que contenha o assunto de resíduos
quadráticos podemos ver a demonstração de que
(-1/p)=1 se p=1 mód 4 e (-1/p)=-1 se
 p=3 mód 4.
Do teorema da reciprocidade quadrática (p é diferente de 2 e 3), temos
(p/3)(3/p)= (-1)^((p-1)/2) -->
Mas p é congruente a 1,5 ,7 ou 11 módulo 12 ( pois é primo e não é 2 ou 3),
testando cada caso, temos (p/3)=1 se p=1 ou 7 módulo 12 (pois devemos ter
p=1 mód 3) e (p/3)=-1 se p=5 ou 11 módulo 12.  Observando que
(-1)^((p-1)/2)=1 se p=1 ou 5 módulo 12 e -1 se p=7 ou 11 módulo 12,
obteremos que (3/p)=1 se p =1 ou 11 módulo 12 e -1 se p =5 ou 7 módulo 12.

Por último, se (-1/p)=(3/p)=1, teremos
p=1 mód 4 e p= 1 ou 11 módulo 12
--> p=1 mód 12.
Se (-1/p)=(3/p)=-1, teremos
p=3 mód 4 e p=5 ou 7 módulo 12
--> p=7 mód 12.
Daí p = 1 ou 7 módulo 12 --> p=1 mód 6, como queríamos.

Em qua, 6 de jun de 2018 11:38, Pedro Chaves 
escreveu:

> Caros Colegas,
>
> Não consegui resolver a questão abaixo. Peço auxílio.
>
> Questão:Para cada inteiro positivo n, mostrar que todo divisor primo
> de 12n^2 + 1 é da forma 6k +1, sendo k um inteiro positivo.
>
>
>
>
> <https://www.avast.com/sig-email?utm_medium=email_source=link_campaign=sig-email_content=webmail>
>  Livre
> de vírus. www.avast.com
> <https://www.avast.com/sig-email?utm_medium=email_source=link_campaign=sig-email_content=webmail>.
> <#m_-594041030565945_DAB4FAD8-2DD7-40BB-A1B8-4E2AA1F9FDF2>
>
> --
> Esta mensagem foi verificada pelo sistema de antivírus e
> acredita-se estar livre de perigo.
>

-- 
Esta mensagem foi verificada pelo sistema de antiv�rus e
 acredita-se estar livre de perigo.



[obm-l] Re: [obm-l] Re: [obm-l] Questão sobre divisor primo

2018-06-06 Por tôpico Claudio Buffara
Na verdade é pra provar que se p é primo e divide 12n^2 + 1, então p é de
forma 6k+1.



2018-06-06 12:50 GMT-03:00 Daniel Quevedo :

> De uma maneira bem informal 6| 12n^2 , para qqr n inteiro. Logo 12n^2+1= 1
> (mod 6) ou seja é da forma 6k +1.
>
> Uma demonstração formal seria por indução finita, onde P(0)= 12+1 = 13 =
> 1(mod 6)
> Se P(n) é verdade, logo
> P (n +1) = 12n^2 + 24n +12 + 1 = 6(2n^2 + 4n + 3) + 13 = 1 (mod 6) é vdd
> Acho q é isso
>
> Em qua, 6 de jun de 2018 às 11:38, Pedro Chaves 
> escreveu:
>
>> Caros Colegas,
>>
>> Não consegui resolver a questão abaixo. Peço auxílio.
>>
>> Questão:Para cada inteiro positivo n, mostrar que todo divisor primo
>> de 12n^2 + 1 é da forma 6k +1, sendo k um inteiro positivo.
>>
>>
>>
>>
>> <https://www.avast.com/sig-email?utm_medium=email_source=link_campaign=sig-email_content=webmail>
>>  Livre
>> de vírus. www.avast.com
>> <https://www.avast.com/sig-email?utm_medium=email_source=link_campaign=sig-email_content=webmail>.
>>
>> <#m_-1183774050225021055_m_7705839601006527736_DAB4FAD8-2DD7-40BB-A1B8-4E2AA1F9FDF2>
>>
>> --
>> Esta mensagem foi verificada pelo sistema de antivírus e
>> acredita-se estar livre de perigo.
>>
> --
> Fiscal: Daniel Quevedo
>
> --
> Esta mensagem foi verificada pelo sistema de antivírus e
> acredita-se estar livre de perigo.
>

-- 
Esta mensagem foi verificada pelo sistema de antiv�rus e
 acredita-se estar livre de perigo.



[obm-l] Re: [obm-l] Questão sobre divisor primo

2018-06-06 Por tôpico Daniel Quevedo
De uma maneira bem informal 6| 12n^2 , para qqr n inteiro. Logo 12n^2+1= 1
(mod 6) ou seja é da forma 6k +1.

Uma demonstração formal seria por indução finita, onde P(0)= 12+1 = 13 =
1(mod 6)
Se P(n) é verdade, logo
P (n +1) = 12n^2 + 24n +12 + 1 = 6(2n^2 + 4n + 3) + 13 = 1 (mod 6) é vdd
Acho q é isso

Em qua, 6 de jun de 2018 às 11:38, Pedro Chaves 
escreveu:

> Caros Colegas,
>
> Não consegui resolver a questão abaixo. Peço auxílio.
>
> Questão:Para cada inteiro positivo n, mostrar que todo divisor primo
> de 12n^2 + 1 é da forma 6k +1, sendo k um inteiro positivo.
>
>
>
>
> <https://www.avast.com/sig-email?utm_medium=email_source=link_campaign=sig-email_content=webmail>
>  Livre
> de vírus. www.avast.com
> <https://www.avast.com/sig-email?utm_medium=email_source=link_campaign=sig-email_content=webmail>.
> <#m_7705839601006527736_DAB4FAD8-2DD7-40BB-A1B8-4E2AA1F9FDF2>
>
> --
> Esta mensagem foi verificada pelo sistema de antivírus e
> acredita-se estar livre de perigo.
>
-- 
Fiscal: Daniel Quevedo

-- 
Esta mensagem foi verificada pelo sistema de antiv�rus e
 acredita-se estar livre de perigo.



[obm-l] Questão sobre divisor primo

2018-06-06 Por tôpico Pedro Chaves
Caros Colegas,

Não consegui resolver a questão abaixo. Peço auxílio.

Questão:Para cada inteiro positivo n, mostrar que todo divisor primo de 
12n^2 + 1 é da forma 6k +1, sendo k um inteiro positivo.



[https://ipmcdn.avast.com/images/icons/icon-envelope-tick-round-orange-animated-no-repeat-v1.gif]<https://www.avast.com/sig-email?utm_medium=email_source=link_campaign=sig-email_content=webmail>
  Livre de vírus. 
www.avast.com<https://www.avast.com/sig-email?utm_medium=email_source=link_campaign=sig-email_content=webmail>.

-- 
Esta mensagem foi verificada pelo sistema de antivírus e
 acredita-se estar livre de perigo.



[obm-l] Re: [obm-l] Re: [obm-l] Re: [obm-l] Re: [obm-l] Questão de derivada

2018-04-23 Por tôpico Igor Caetano Diniz
na verdade eu não fiz rsrs.

Eu queria ver um modo claro de mostrar. Se não puder usar L'Hospital, acho
que tem que fazer uma sequência por baixo e uma por cima aplicando TVM em
cada intervalo. Aí usa o fato dessa sequencia ser limitada, e monotona,
portanto, convergente. Logo lim f'(xn) = L tanto por cima, quanto por
baixo. como existem esses limites laterais e de mesmo valor, a derivada
existe.

-- 
Esta mensagem foi verificada pelo sistema de antiv�rus e
 acredita-se estar livre de perigo.



[obm-l] Re: [obm-l] Re: [obm-l] Re: [obm-l] Questão de derivada

2018-04-23 Por tôpico Artur Steiner
Eu li errado, temos que lim x --> 0 f'
(x) = L. Assim,  a Regra de l' Hopital conforme mostrei demonstra que, de
fato, f'(c) = L.

Mas o que vc fez não mostra que f'(c) = L.

Artur Costa Steiner

Em Seg, 23 de abr de 2018 14:31, Igor Caetano Diniz <icaetanodi...@gmail.com>
escreveu:

> Se a questão tivesse um intervalo explícito [a,b] e diferenciável em todo
> ponto (a,b) exceto possivelmente num ponto c em (a,b) tal que lim f '(x) =
> L, x-> c, o que eu fiz estaria correto?
>
> 2018-04-23 14:11 GMT-03:00 Artur Steiner <artur.costa.stei...@gmail.com>:
>
>> Como f é contínua em 0, então, pela regra de L'Hopital,
>>
>> lim x --> 0+  (f(x) - f0))/(x - 0) = lim x --> 0+ f'(x) = L
>>
>> Pela definição de derivada lateral, o limite do primeiro membro é a
>> derivada à direita de 0. É só o que podemos concluir do enunciado. Nada
>> garante que a derivada à esquerda de 0 sequer exista.
>>
>> Artur Costa Steiner
>>
>> Em Dom, 22 de abr de 2018 22:45, Igor Caetano Diniz <
>> icaetanodi...@gmail.com> escreveu:
>>
>>> Boa noite,
>>> Gostaria de uma ajuda numa questão. Primeiro saber se pensei
>>> corretamente na maneira (1) e se é possível resolver como pensei também na
>>> maneira (2).
>>> Aí vai:
>>> Questão 5.3.8 do livro do Stephen Abbot, Understanding Analysis:
>>>
>>> Assuma que f é contínua em um intervalo que contém o zero e
>>> diferenciável em todo ponto diferente de zero. Se lim f ' (x) = L, x->0,
>>> prove que f ' (0) existe e é igual a L.
>>>
>>> O que pensei em fazer:
>>>
>>> Pensei em duas maneiras.
>>> 1)Se o limite existe em 0, então existem os limites laterais, limite a
>>> esquerda e limite a direita: lim x->0- f ' (x) = L e lim x->0+ f ' (x) = L.
>>> Lema: f ' (c) = lim f(c+h)-f(c-h)/2h = lim [ f(c+h)-f(c) +f(c) - f(c-h)
>>> ]/2h = 1/2 lim x->c-[f(c+h)-f(c)/h] + 1/2 lim x->c+ [f(c+h)-f(c)/h]
>>>
>>> Logo como existem esses limites laterais, existe a derivada em 0, e
>>> portanto, é L
>>>
>>> 2) queria tentar fazer, usando uma sequência xn<0 com limxn = 0 e yn>0
>>> com lim yn = 0 e provar que lim(f(yn)-f(xn)/(yn-xn)) = f'(0) = L. Mas sinto
>>> que isso é verdade e não sei provar
>>>
>>> Abraços
>>>
>>> --
>>> Esta mensagem foi verificada pelo sistema de antivírus e
>>> acredita-se estar livre de perigo.
>>
>>
>> --
>> Esta mensagem foi verificada pelo sistema de antivírus e
>> acredita-se estar livre de perigo.
>>
>
>
> --
> Esta mensagem foi verificada pelo sistema de antivírus e
> acredita-se estar livre de perigo.

-- 
Esta mensagem foi verificada pelo sistema de antiv�rus e
 acredita-se estar livre de perigo.



[obm-l] Re: [obm-l] Re: [obm-l] Questão de derivada

2018-04-23 Por tôpico Igor Caetano Diniz
Se a questão tivesse um intervalo explícito [a,b] e diferenciável em todo
ponto (a,b) exceto possivelmente num ponto c em (a,b) tal que lim f '(x) =
L, x-> c, o que eu fiz estaria correto?

2018-04-23 14:11 GMT-03:00 Artur Steiner <artur.costa.stei...@gmail.com>:

> Como f é contínua em 0, então, pela regra de L'Hopital,
>
> lim x --> 0+  (f(x) - f0))/(x - 0) = lim x --> 0+ f'(x) = L
>
> Pela definição de derivada lateral, o limite do primeiro membro é a
> derivada à direita de 0. É só o que podemos concluir do enunciado. Nada
> garante que a derivada à esquerda de 0 sequer exista.
>
> Artur Costa Steiner
>
> Em Dom, 22 de abr de 2018 22:45, Igor Caetano Diniz <
> icaetanodi...@gmail.com> escreveu:
>
>> Boa noite,
>> Gostaria de uma ajuda numa questão. Primeiro saber se pensei corretamente
>> na maneira (1) e se é possível resolver como pensei também na maneira (2).
>> Aí vai:
>> Questão 5.3.8 do livro do Stephen Abbot, Understanding Analysis:
>>
>> Assuma que f é contínua em um intervalo que contém o zero e diferenciável
>> em todo ponto diferente de zero. Se lim f ' (x) = L, x->0, prove que f '
>> (0) existe e é igual a L.
>>
>> O que pensei em fazer:
>>
>> Pensei em duas maneiras.
>> 1)Se o limite existe em 0, então existem os limites laterais, limite a
>> esquerda e limite a direita: lim x->0- f ' (x) = L e lim x->0+ f ' (x) = L.
>> Lema: f ' (c) = lim f(c+h)-f(c-h)/2h = lim [ f(c+h)-f(c) +f(c) - f(c-h)
>> ]/2h = 1/2 lim x->c-[f(c+h)-f(c)/h] + 1/2 lim x->c+ [f(c+h)-f(c)/h]
>>
>> Logo como existem esses limites laterais, existe a derivada em 0, e
>> portanto, é L
>>
>> 2) queria tentar fazer, usando uma sequência xn<0 com limxn = 0 e yn>0
>> com lim yn = 0 e provar que lim(f(yn)-f(xn)/(yn-xn)) = f'(0) = L. Mas sinto
>> que isso é verdade e não sei provar
>>
>> Abraços
>>
>> --
>> Esta mensagem foi verificada pelo sistema de antivírus e
>> acredita-se estar livre de perigo.
>
>
> --
> Esta mensagem foi verificada pelo sistema de antivírus e
> acredita-se estar livre de perigo.
>

-- 
Esta mensagem foi verificada pelo sistema de antiv�rus e
 acredita-se estar livre de perigo.



[obm-l] Re: [obm-l] Questão de derivada

2018-04-23 Por tôpico Artur Steiner
Como f é contínua em 0, então, pela regra de L'Hopital,

lim x --> 0+  (f(x) - f0))/(x - 0) = lim x --> 0+ f'(x) = L

Pela definição de derivada lateral, o limite do primeiro membro é a
derivada à direita de 0. É só o que podemos concluir do enunciado. Nada
garante que a derivada à esquerda de 0 sequer exista.

Artur Costa Steiner

Em Dom, 22 de abr de 2018 22:45, Igor Caetano Diniz <icaetanodi...@gmail.com>
escreveu:

> Boa noite,
> Gostaria de uma ajuda numa questão. Primeiro saber se pensei corretamente
> na maneira (1) e se é possível resolver como pensei também na maneira (2).
> Aí vai:
> Questão 5.3.8 do livro do Stephen Abbot, Understanding Analysis:
>
> Assuma que f é contínua em um intervalo que contém o zero e diferenciável
> em todo ponto diferente de zero. Se lim f ' (x) = L, x->0, prove que f '
> (0) existe e é igual a L.
>
> O que pensei em fazer:
>
> Pensei em duas maneiras.
> 1)Se o limite existe em 0, então existem os limites laterais, limite a
> esquerda e limite a direita: lim x->0- f ' (x) = L e lim x->0+ f ' (x) = L.
> Lema: f ' (c) = lim f(c+h)-f(c-h)/2h = lim [ f(c+h)-f(c) +f(c) - f(c-h)
> ]/2h = 1/2 lim x->c-[f(c+h)-f(c)/h] + 1/2 lim x->c+ [f(c+h)-f(c)/h]
>
> Logo como existem esses limites laterais, existe a derivada em 0, e
> portanto, é L
>
> 2) queria tentar fazer, usando uma sequência xn<0 com limxn = 0 e yn>0 com
> lim yn = 0 e provar que lim(f(yn)-f(xn)/(yn-xn)) = f'(0) = L. Mas sinto que
> isso é verdade e não sei provar
>
> Abraços
>
> --
> Esta mensagem foi verificada pelo sistema de antivírus e
> acredita-se estar livre de perigo.

-- 
Esta mensagem foi verificada pelo sistema de antiv�rus e
 acredita-se estar livre de perigo.



[obm-l] Re: [obm-l] Re: [obm-l] Questão de derivada

2018-04-23 Por tôpico Igor Caetano Diniz
Então,

Se existem os limites laterais, lim f ' (0-) = lim f ' (0+) então, defina
q(x) = [f(x) - f(0)]/x. Para todo x<0, existe y1 entre x e 0 tal que f '
(y) = q(x). Analogamente para x>0, existe z1 entre 0 e x tal que f ' (z) =
q(x).
Defina r(x,0) a distancia de x para 0
Então, seja yn = yn-1 + r(y_n-1,0)/2 e zn = zn-1 + r(z_n-1,0)/2. Tal
sequência converge para o 0 com lim yn = 0 = lim zn. Além disso, lim f '
(yn) = L = lim f ' (zn)

Não estou conseguindo concluir. Alguém poderia ajudar?

Abraços

2018-04-23 11:25 GMT-03:00 Bernardo Freitas Paulo da Costa <
bernardo...@gmail.com>:

> 2018-04-22 22:36 GMT-03:00 Igor Caetano Diniz <icaetanodi...@gmail.com>:
> > Boa noite,
> > Gostaria de uma ajuda numa questão. Primeiro saber se pensei
> corretamente na
> > maneira (1) e se é possível resolver como pensei também na maneira (2).
> > Aí vai:
> > Questão 5.3.8 do livro do Stephen Abbot, Understanding Analysis:
> >
> > Assuma que f é contínua em um intervalo que contém o zero e
> diferenciável em
> > todo ponto diferente de zero. Se lim f ' (x) = L, x->0, prove que f ' (0)
> > existe e é igual a L.
> >
> > O que pensei em fazer:
> >
> > Pensei em duas maneiras.
> > 1)Se o limite existe em 0, então existem os limites laterais, limite a
> > esquerda e limite a direita: lim x->0- f ' (x) = L e lim x->0+ f ' (x) =
> L.
> > Lema: f ' (c) = lim f(c+h)-f(c-h)/2h = lim [ f(c+h)-f(c) +f(c) - f(c-h)
> ]/2h
> > = 1/2 lim x->c-[f(c+h)-f(c)/h] + 1/2 lim x->c+ [f(c+h)-f(c)/h]
> >
> > Logo como existem esses limites laterais, existe a derivada em 0, e
> > portanto, é L
>
> Cuidado, Igor: a existência do limite [f(c+h) - f(c-h)]/2h quando h ->
> 0 não implica que existe a derivada.  Por exemplo, se f(x) = |x|, o
> limite dá zero, mas a derivada não existe.
>
> > 2) queria tentar fazer, usando uma sequência xn<0 com limxn = 0 e yn>0
> com
> > lim yn = 0 e provar que lim(f(yn)-f(xn)/(yn-xn)) = f'(0) = L. Mas sinto
> que
> > isso é verdade e não sei provar
>
> De novo, pelo mesmo exemplo acima, não basta provar que os limites com
> um ponto de cada lado dão certo.  Acho (só acho) que se *todos* os
> limites possíveis derem iguais, então a derivada existe, o que
> justifica a sua abordagem, mas daí você teria que provar o enunciado
> geral
>
> "Se f é contínua, e para TODA sequência x_n < 0 < y_n, com x_n -> 0,
> y_n ->0, vale que [f(y_n) - f(x_n)]/(y_n - x_n) -> L, então f é
> derivável em zero, e f'(0) = L."
>
> Abraços,
> --
> Bernardo Freitas Paulo da Costa
>
> --
> Esta mensagem foi verificada pelo sistema de antivírus e
>  acredita-se estar livre de perigo.
>
>
> =
> Instru�ões para entrar na lista, sair da lista e usar a lista em
> http://www.mat.puc-rio.br/~obmlistas/obm-l.html
> =
>

-- 
Esta mensagem foi verificada pelo sistema de antiv�rus e
 acredita-se estar livre de perigo.



[obm-l] Re: [obm-l] Questão de derivada

2018-04-23 Por tôpico Bernardo Freitas Paulo da Costa
2018-04-22 22:36 GMT-03:00 Igor Caetano Diniz <icaetanodi...@gmail.com>:
> Boa noite,
> Gostaria de uma ajuda numa questão. Primeiro saber se pensei corretamente na
> maneira (1) e se é possível resolver como pensei também na maneira (2).
> Aí vai:
> Questão 5.3.8 do livro do Stephen Abbot, Understanding Analysis:
>
> Assuma que f é contínua em um intervalo que contém o zero e diferenciável em
> todo ponto diferente de zero. Se lim f ' (x) = L, x->0, prove que f ' (0)
> existe e é igual a L.
>
> O que pensei em fazer:
>
> Pensei em duas maneiras.
> 1)Se o limite existe em 0, então existem os limites laterais, limite a
> esquerda e limite a direita: lim x->0- f ' (x) = L e lim x->0+ f ' (x) = L.
> Lema: f ' (c) = lim f(c+h)-f(c-h)/2h = lim [ f(c+h)-f(c) +f(c) - f(c-h) ]/2h
> = 1/2 lim x->c-[f(c+h)-f(c)/h] + 1/2 lim x->c+ [f(c+h)-f(c)/h]
>
> Logo como existem esses limites laterais, existe a derivada em 0, e
> portanto, é L

Cuidado, Igor: a existência do limite [f(c+h) - f(c-h)]/2h quando h ->
0 não implica que existe a derivada.  Por exemplo, se f(x) = |x|, o
limite dá zero, mas a derivada não existe.

> 2) queria tentar fazer, usando uma sequência xn<0 com limxn = 0 e yn>0 com
> lim yn = 0 e provar que lim(f(yn)-f(xn)/(yn-xn)) = f'(0) = L. Mas sinto que
> isso é verdade e não sei provar

De novo, pelo mesmo exemplo acima, não basta provar que os limites com
um ponto de cada lado dão certo.  Acho (só acho) que se *todos* os
limites possíveis derem iguais, então a derivada existe, o que
justifica a sua abordagem, mas daí você teria que provar o enunciado
geral

"Se f é contínua, e para TODA sequência x_n < 0 < y_n, com x_n -> 0,
y_n ->0, vale que [f(y_n) - f(x_n)]/(y_n - x_n) -> L, então f é
derivável em zero, e f'(0) = L."

Abraços,
-- 
Bernardo Freitas Paulo da Costa

-- 
Esta mensagem foi verificada pelo sistema de antiv�rus e
 acredita-se estar livre de perigo.


=
Instru��es para entrar na lista, sair da lista e usar a lista em
http://www.mat.puc-rio.br/~obmlistas/obm-l.html
=


[obm-l] Questão de derivada

2018-04-22 Por tôpico Igor Caetano Diniz
Boa noite,
Gostaria de uma ajuda numa questão. Primeiro saber se pensei corretamente
na maneira (1) e se é possível resolver como pensei também na maneira (2).
Aí vai:
Questão 5.3.8 do livro do Stephen Abbot, Understanding Analysis:

Assuma que f é contínua em um intervalo que contém o zero e diferenciável
em todo ponto diferente de zero. Se lim f ' (x) = L, x->0, prove que f '
(0) existe e é igual a L.

O que pensei em fazer:

Pensei em duas maneiras.
1)Se o limite existe em 0, então existem os limites laterais, limite a
esquerda e limite a direita: lim x->0- f ' (x) = L e lim x->0+ f ' (x) = L.
Lema: f ' (c) = lim f(c+h)-f(c-h)/2h = lim [ f(c+h)-f(c) +f(c) - f(c-h)
]/2h = 1/2 lim x->c-[f(c+h)-f(c)/h] + 1/2 lim x->c+ [f(c+h)-f(c)/h]

Logo como existem esses limites laterais, existe a derivada em 0, e
portanto, é L

2) queria tentar fazer, usando uma sequência xn<0 com limxn = 0 e yn>0 com
lim yn = 0 e provar que lim(f(yn)-f(xn)/(yn-xn)) = f'(0) = L. Mas sinto que
isso é verdade e não sei provar

Abraços

-- 
Esta mensagem foi verificada pelo sistema de antiv�rus e
 acredita-se estar livre de perigo.



[obm-l] Re: [obm-l] Re: [obm-l] Re: [obm-l] Re: [obm-l] Questão de Combinatória

2018-03-30 Por tôpico Anderson Torres
Em 29 de março de 2018 15:37, Igor Caetano Diniz
 escreveu:
> Vou mostrar a sua e a minha e aí se ele não aprender com as duas, tento
> fazer devagar em casos menores. hehe
>
> Abraços Cláudio e obrigado =)
>
> 2018-03-29 15:17 GMT-03:00 Claudio Buffara :
>>
>> Sim. Acho essa uma solução bem mais elegante.
>> Mas também é mais sofisticada, e você falou que o aluno é principiante.
>>
>> De todo jeito, acho que raciocinar recursivamente é uma habilidade que
>> todo estudante de matemática deveria desenvolver.
>>

Não seria mais interessante ir "montando" as possíveis sequências num
diagrama de árvore mesmo, já que é para fazer no bração?

Cada nível da árvore é obtido acrescentando 0 e 1 ao final do nível
anterior e aniquilando os que quebram o padrão (dois 1s consecutivos).

*

0 1

00 01 10 11X

000 001 010 011X 100 101

 0001 0010 0011X 0100 0101 1000 1001 1010 1011X

E assim por diante. Afinal, se é para contar na mão, tem que organizar.

>> []s,
>> Claudio.
>>
>>
>> 2018-03-29 14:45 GMT-03:00 Igor Caetano Diniz :
>>>
>>> Olá Claudio
>>> Pensei numa solução agora que acredito que eu possa explicar e a pessoa
>>> irá entender:
>>>
>>> Para 1 bit, 2 possibilidades
>>> Para 2 bits, 3
>>> Para 3 bits, basta separar em casos: Se for 0 _ _, cai no caso anterior.
>>> Se for 1 _ _ tem que ser  1 0 _ e, então, cai no caso anterior-1.
>>> Para 4 bits, separe de novo: 0 _ _ _, que cai no problema anterior, ou, 1
>>> 0 _ _, caindo no anterior -1.
>>> Ou seja, Para N bits: F(N) = F(N-1) + F(N-2). É um Fibonacci começando de
>>> F(1) = 2 e F(2) = 3
>>>
>>>
>>> Estaria correto assim?
>>>
>>> Abraços
>>>
>>> 2018-03-29 14:26 GMT-03:00 Claudio Buffara :

 Sugestão: separe em casos em função do número N de 1’s na sequência.

 N = 0: 1 sequência
 N = 1: 8 sequências
 N = 2: 8*7/2 - 7 = 21
 (No de sequências sem restrições menos o no de sequências com os dois
 1’s adjacentes)
 N = 4: 2
 N > 4: 0

 O caso N = 3 é o mais chatinho pois tem mais subcasos, mas não chega a
 ser difícil.

 Depois eu mando.

 Abs





 Enviado do meu iPhone

 Em 29 de mar de 2018, à(s) 13:31, Igor Caetano Diniz
  escreveu:

 > Olá pessoal,
 >
 > Estou com uma questão de Combinatória e gostaria de uma solução
 > didática para ela pq como eu fiz ficou complexo para um aluno que 
 > iniciou
 > combinatória agora.
 > segue a questão:
 >
 > Quantas sequências de 8 bits(com 0's e 1's) não têm dois 1
 > consecutivos?
 >
 > Como foi resolvida: usando variáveis para contar quantos 0 estão
 > entre 1's consecutivos, separada em casos de dois, três e quatro 1's
 > consecutivos. Mas assim fica difícil para quem começou a aprender 
 > agora.
 >
 > Abraços
 >
 > --
 > Esta mensagem foi verificada pelo sistema de antivírus e
 > acredita-se estar livre de perigo.

 --
 Esta mensagem foi verificada pelo sistema de antivírus e
  acredita-se estar livre de perigo.



 =
 Instru�ões para entrar na lista, sair da lista e usar a lista em
 http://www.mat.puc-rio.br/~obmlistas/obm-l.html

 =
>>>
>>>
>>>
>>> --
>>> Esta mensagem foi verificada pelo sistema de antivírus e
>>> acredita-se estar livre de perigo.
>>
>>
>>
>> --
>> Esta mensagem foi verificada pelo sistema de antivírus e
>> acredita-se estar livre de perigo.
>
>
>
> --
> Esta mensagem foi verificada pelo sistema de antivírus e
> acredita-se estar livre de perigo.

-- 
Esta mensagem foi verificada pelo sistema de antiv�rus e
 acredita-se estar livre de perigo.


=
Instru��es para entrar na lista, sair da lista e usar a lista em
http://www.mat.puc-rio.br/~obmlistas/obm-l.html
=


[obm-l] Re: [obm-l] Re: [obm-l] Re: [obm-l] Re: [obm-l] Questão de Combinatória

2018-03-29 Por tôpico Claudio Buffara
Outra sugestão: proponha o problema de contar de quantas maneiras é
possível arrumar N dominós 1x2 numa caixa 2xN.
Fibonacci também aparece neste aí.

A diferença é que, no dos bits, B(N) = F(N+2) enquanto que, no dos dominós,
D(N) = F(N+1)
(F é definida da forma usual, com F(1) = F(2) = 1)

Ou então: quantas sequências de 1's e 2's existem que têm soma N?
Aqui, X(N) = F(N+1) também.

Um problema complementar interessante é achar bijeções "naturais" entre as
sequências definidas por estes três problemas.
Entre D e X é fácil.  Entre estes e as suas sequências de bits nem tanto.

[]s,
Claudio.

-- 
Esta mensagem foi verificada pelo sistema de antiv�rus e
 acredita-se estar livre de perigo.



[obm-l] Re: [obm-l] Re: [obm-l] Questão de Combinatória

2018-03-29 Por tôpico Claudio Buffara
Sugestão de natureza didática: eu mostraria uma solução mais braçal, tal
como a minha, e depois mostraria a solução recursiva.
Moral: em geral vale a pena pensar no problema antes de sair escrevendo...

2018-03-29 15:17 GMT-03:00 Claudio Buffara :

> Sim. Acho essa uma solução bem mais elegante.
> Mas também é mais sofisticada, e você falou que o aluno é principiante.
>
> De todo jeito, acho que raciocinar recursivamente é uma habilidade que
> todo estudante de matemática deveria desenvolver.
>
> []s,
> Claudio.
>
>
> 2018-03-29 14:45 GMT-03:00 Igor Caetano Diniz :
>
>> Olá Claudio
>> Pensei numa solução agora que acredito que eu possa explicar e a pessoa
>> irá entender:
>>
>> Para 1 bit, 2 possibilidades
>> Para 2 bits, 3
>> Para 3 bits, basta separar em casos: Se for 0 _ _, cai no caso anterior.
>> Se for 1 _ _ tem que ser  1 0 _ e, então, cai no caso anterior-1.
>> Para 4 bits, separe de novo: 0 _ _ _, que cai no problema anterior, ou, 1
>> 0 _ _, caindo no anterior -1.
>> Ou seja, Para N bits: F(N) = F(N-1) + F(N-2). É um Fibonacci começando de
>> F(1) = 2 e F(2) = 3
>>
>>
>> Estaria correto assim?
>>
>> Abraços
>>
>> 2018-03-29 14:26 GMT-03:00 Claudio Buffara :
>>
>>> Sugestão: separe em casos em função do número N de 1’s na sequência.
>>>
>>> N = 0: 1 sequência
>>> N = 1: 8 sequências
>>> N = 2: 8*7/2 - 7 = 21
>>> (No de sequências sem restrições menos o no de sequências com os dois
>>> 1’s adjacentes)
>>> N = 4: 2
>>> N > 4: 0
>>>
>>> O caso N = 3 é o mais chatinho pois tem mais subcasos, mas não chega a
>>> ser difícil.
>>>
>>> Depois eu mando.
>>>
>>> Abs
>>>
>>>
>>>
>>>
>>>
>>> Enviado do meu iPhone
>>>
>>> Em 29 de mar de 2018, à(s) 13:31, Igor Caetano Diniz <
>>> icaetanodi...@gmail.com> escreveu:
>>>
>>> > Olá pessoal,
>>> >
>>> > Estou com uma questão de Combinatória e gostaria de uma solução
>>> didática para ela pq como eu fiz ficou complexo para um aluno que iniciou
>>> combinatória agora.
>>> > segue a questão:
>>> >
>>> > Quantas sequências de 8 bits(com 0's e 1's) não têm dois 1
>>> consecutivos?
>>> >
>>> > Como foi resolvida: usando variáveis para contar quantos 0 estão
>>> entre 1's consecutivos, separada em casos de dois, três e quatro 1's
>>> consecutivos. Mas assim fica difícil para quem começou a aprender agora.
>>> >
>>> > Abraços
>>> >
>>> > --
>>> > Esta mensagem foi verificada pelo sistema de antivírus e
>>> > acredita-se estar livre de perigo.
>>>
>>> --
>>> Esta mensagem foi verificada pelo sistema de antivírus e
>>>  acredita-se estar livre de perigo.
>>>
>>>
>>> 
>>> =
>>> Instru�ões para entrar na lista, sair da lista e usar a lista em
>>> http://www.mat.puc-rio.br/~obmlistas/obm-l.html
>>> 
>>> =
>>>
>>
>>
>> --
>> Esta mensagem foi verificada pelo sistema de antivírus e
>> acredita-se estar livre de perigo.
>>
>
>

-- 
Esta mensagem foi verificada pelo sistema de antiv�rus e
 acredita-se estar livre de perigo.



[obm-l] Re: [obm-l] Re: [obm-l] Re: [obm-l] Questão de Combinatória

2018-03-29 Por tôpico Igor Caetano Diniz
Vou mostrar a sua e a minha e aí se ele não aprender com as duas, tento
fazer devagar em casos menores. hehe

Abraços Cláudio e obrigado =)

2018-03-29 15:17 GMT-03:00 Claudio Buffara :

> Sim. Acho essa uma solução bem mais elegante.
> Mas também é mais sofisticada, e você falou que o aluno é principiante.
>
> De todo jeito, acho que raciocinar recursivamente é uma habilidade que
> todo estudante de matemática deveria desenvolver.
>
> []s,
> Claudio.
>
>
> 2018-03-29 14:45 GMT-03:00 Igor Caetano Diniz :
>
>> Olá Claudio
>> Pensei numa solução agora que acredito que eu possa explicar e a pessoa
>> irá entender:
>>
>> Para 1 bit, 2 possibilidades
>> Para 2 bits, 3
>> Para 3 bits, basta separar em casos: Se for 0 _ _, cai no caso anterior.
>> Se for 1 _ _ tem que ser  1 0 _ e, então, cai no caso anterior-1.
>> Para 4 bits, separe de novo: 0 _ _ _, que cai no problema anterior, ou, 1
>> 0 _ _, caindo no anterior -1.
>> Ou seja, Para N bits: F(N) = F(N-1) + F(N-2). É um Fibonacci começando de
>> F(1) = 2 e F(2) = 3
>>
>>
>> Estaria correto assim?
>>
>> Abraços
>>
>> 2018-03-29 14:26 GMT-03:00 Claudio Buffara :
>>
>>> Sugestão: separe em casos em função do número N de 1’s na sequência.
>>>
>>> N = 0: 1 sequência
>>> N = 1: 8 sequências
>>> N = 2: 8*7/2 - 7 = 21
>>> (No de sequências sem restrições menos o no de sequências com os dois
>>> 1’s adjacentes)
>>> N = 4: 2
>>> N > 4: 0
>>>
>>> O caso N = 3 é o mais chatinho pois tem mais subcasos, mas não chega a
>>> ser difícil.
>>>
>>> Depois eu mando.
>>>
>>> Abs
>>>
>>>
>>>
>>>
>>>
>>> Enviado do meu iPhone
>>>
>>> Em 29 de mar de 2018, à(s) 13:31, Igor Caetano Diniz <
>>> icaetanodi...@gmail.com> escreveu:
>>>
>>> > Olá pessoal,
>>> >
>>> > Estou com uma questão de Combinatória e gostaria de uma solução
>>> didática para ela pq como eu fiz ficou complexo para um aluno que iniciou
>>> combinatória agora.
>>> > segue a questão:
>>> >
>>> > Quantas sequências de 8 bits(com 0's e 1's) não têm dois 1
>>> consecutivos?
>>> >
>>> > Como foi resolvida: usando variáveis para contar quantos 0 estão
>>> entre 1's consecutivos, separada em casos de dois, três e quatro 1's
>>> consecutivos. Mas assim fica difícil para quem começou a aprender agora.
>>> >
>>> > Abraços
>>> >
>>> > --
>>> > Esta mensagem foi verificada pelo sistema de antivírus e
>>> > acredita-se estar livre de perigo.
>>>
>>> --
>>> Esta mensagem foi verificada pelo sistema de antivírus e
>>>  acredita-se estar livre de perigo.
>>>
>>>
>>> 
>>> =
>>> Instru�ões para entrar na lista, sair da lista e usar a lista em
>>> http://www.mat.puc-rio.br/~obmlistas/obm-l.html
>>> 
>>> =
>>>
>>
>>
>> --
>> Esta mensagem foi verificada pelo sistema de antivírus e
>> acredita-se estar livre de perigo.
>>
>
>
> --
> Esta mensagem foi verificada pelo sistema de antivírus e
> acredita-se estar livre de perigo.
>

-- 
Esta mensagem foi verificada pelo sistema de antiv�rus e
 acredita-se estar livre de perigo.



[obm-l] Re: [obm-l] Re: [obm-l] Questão de Combinatória

2018-03-29 Por tôpico Claudio Buffara
Sim. Acho essa uma solução bem mais elegante.
Mas também é mais sofisticada, e você falou que o aluno é principiante.

De todo jeito, acho que raciocinar recursivamente é uma habilidade que todo
estudante de matemática deveria desenvolver.

[]s,
Claudio.


2018-03-29 14:45 GMT-03:00 Igor Caetano Diniz :

> Olá Claudio
> Pensei numa solução agora que acredito que eu possa explicar e a pessoa
> irá entender:
>
> Para 1 bit, 2 possibilidades
> Para 2 bits, 3
> Para 3 bits, basta separar em casos: Se for 0 _ _, cai no caso anterior.
> Se for 1 _ _ tem que ser  1 0 _ e, então, cai no caso anterior-1.
> Para 4 bits, separe de novo: 0 _ _ _, que cai no problema anterior, ou, 1
> 0 _ _, caindo no anterior -1.
> Ou seja, Para N bits: F(N) = F(N-1) + F(N-2). É um Fibonacci começando de
> F(1) = 2 e F(2) = 3
>
>
> Estaria correto assim?
>
> Abraços
>
> 2018-03-29 14:26 GMT-03:00 Claudio Buffara :
>
>> Sugestão: separe em casos em função do número N de 1’s na sequência.
>>
>> N = 0: 1 sequência
>> N = 1: 8 sequências
>> N = 2: 8*7/2 - 7 = 21
>> (No de sequências sem restrições menos o no de sequências com os dois 1’s
>> adjacentes)
>> N = 4: 2
>> N > 4: 0
>>
>> O caso N = 3 é o mais chatinho pois tem mais subcasos, mas não chega a
>> ser difícil.
>>
>> Depois eu mando.
>>
>> Abs
>>
>>
>>
>>
>>
>> Enviado do meu iPhone
>>
>> Em 29 de mar de 2018, à(s) 13:31, Igor Caetano Diniz <
>> icaetanodi...@gmail.com> escreveu:
>>
>> > Olá pessoal,
>> >
>> > Estou com uma questão de Combinatória e gostaria de uma solução
>> didática para ela pq como eu fiz ficou complexo para um aluno que iniciou
>> combinatória agora.
>> > segue a questão:
>> >
>> > Quantas sequências de 8 bits(com 0's e 1's) não têm dois 1
>> consecutivos?
>> >
>> > Como foi resolvida: usando variáveis para contar quantos 0 estão
>> entre 1's consecutivos, separada em casos de dois, três e quatro 1's
>> consecutivos. Mas assim fica difícil para quem começou a aprender agora.
>> >
>> > Abraços
>> >
>> > --
>> > Esta mensagem foi verificada pelo sistema de antivírus e
>> > acredita-se estar livre de perigo.
>>
>> --
>> Esta mensagem foi verificada pelo sistema de antivírus e
>>  acredita-se estar livre de perigo.
>>
>>
>> =
>> Instru�ões para entrar na lista, sair da lista e usar a lista em
>> http://www.mat.puc-rio.br/~obmlistas/obm-l.html
>> =
>>
>
>
> --
> Esta mensagem foi verificada pelo sistema de antivírus e
> acredita-se estar livre de perigo.
>

-- 
Esta mensagem foi verificada pelo sistema de antiv�rus e
 acredita-se estar livre de perigo.



[obm-l] Re: [obm-l] Questão de Combinatória

2018-03-29 Por tôpico Igor Caetano Diniz
Olá Claudio
Pensei numa solução agora que acredito que eu possa explicar e a pessoa irá
entender:

Para 1 bit, 2 possibilidades
Para 2 bits, 3
Para 3 bits, basta separar em casos: Se for 0 _ _, cai no caso anterior. Se
for 1 _ _ tem que ser  1 0 _ e, então, cai no caso anterior-1.
Para 4 bits, separe de novo: 0 _ _ _, que cai no problema anterior, ou, 1 0
_ _, caindo no anterior -1.
Ou seja, Para N bits: F(N) = F(N-1) + F(N-2). É um Fibonacci começando de
F(1) = 2 e F(2) = 3


Estaria correto assim?

Abraços

2018-03-29 14:26 GMT-03:00 Claudio Buffara :

> Sugestão: separe em casos em função do número N de 1’s na sequência.
>
> N = 0: 1 sequência
> N = 1: 8 sequências
> N = 2: 8*7/2 - 7 = 21
> (No de sequências sem restrições menos o no de sequências com os dois 1’s
> adjacentes)
> N = 4: 2
> N > 4: 0
>
> O caso N = 3 é o mais chatinho pois tem mais subcasos, mas não chega a ser
> difícil.
>
> Depois eu mando.
>
> Abs
>
>
>
>
>
> Enviado do meu iPhone
>
> Em 29 de mar de 2018, à(s) 13:31, Igor Caetano Diniz <
> icaetanodi...@gmail.com> escreveu:
>
> > Olá pessoal,
> >
> > Estou com uma questão de Combinatória e gostaria de uma solução
> didática para ela pq como eu fiz ficou complexo para um aluno que iniciou
> combinatória agora.
> > segue a questão:
> >
> > Quantas sequências de 8 bits(com 0's e 1's) não têm dois 1
> consecutivos?
> >
> > Como foi resolvida: usando variáveis para contar quantos 0 estão entre
> 1's consecutivos, separada em casos de dois, três e quatro 1's
> consecutivos. Mas assim fica difícil para quem começou a aprender agora.
> >
> > Abraços
> >
> > --
> > Esta mensagem foi verificada pelo sistema de antivírus e
> > acredita-se estar livre de perigo.
>
> --
> Esta mensagem foi verificada pelo sistema de antivírus e
>  acredita-se estar livre de perigo.
>
>
> =
> Instru�ões para entrar na lista, sair da lista e usar a lista em
> http://www.mat.puc-rio.br/~obmlistas/obm-l.html
> =
>

-- 
Esta mensagem foi verificada pelo sistema de antiv�rus e
 acredita-se estar livre de perigo.



Re: [obm-l] Questão de Combinatória

2018-03-29 Por tôpico Claudio Buffara
Sugestão: separe em casos em função do número N de 1’s na sequência.

N = 0: 1 sequência 
N = 1: 8 sequências
N = 2: 8*7/2 - 7 = 21 
(No de sequências sem restrições menos o no de sequências com os dois 1’s 
adjacentes)
N = 4: 2
N > 4: 0

O caso N = 3 é o mais chatinho pois tem mais subcasos, mas não chega a ser 
difícil.

Depois eu mando.

Abs 





Enviado do meu iPhone

Em 29 de mar de 2018, à(s) 13:31, Igor Caetano Diniz  
escreveu:

> Olá pessoal,
> 
> Estou com uma questão de Combinatória e gostaria de uma solução didática 
> para ela pq como eu fiz ficou complexo para um aluno que iniciou 
> combinatória agora.
> segue a questão:
> 
> Quantas sequências de 8 bits(com 0's e 1's) não têm dois 1 consecutivos?
> 
> Como foi resolvida: usando variáveis para contar quantos 0 estão entre 1's 
> consecutivos, separada em casos de dois, três e quatro 1's consecutivos. Mas 
> assim fica difícil para quem começou a aprender agora.
> 
> Abraços
> 
> -- 
> Esta mensagem foi verificada pelo sistema de antivírus e 
> acredita-se estar livre de perigo.

-- 
Esta mensagem foi verificada pelo sistema de antiv�rus e
 acredita-se estar livre de perigo.


=
Instru��es para entrar na lista, sair da lista e usar a lista em
http://www.mat.puc-rio.br/~obmlistas/obm-l.html
=


[obm-l] Questão de Combinatória

2018-03-29 Por tôpico Igor Caetano Diniz
Olá pessoal,

Estou com uma questão de Combinatória e gostaria de uma solução didática
para ela pq como eu fiz ficou complexo para um aluno que iniciou
combinatória agora.
segue a questão:

Quantas sequências de 8 bits(com 0's e 1's) não têm dois 1 consecutivos?

Como foi resolvida: usando variáveis para contar quantos 0 estão entre 1's
consecutivos, separada em casos de dois, três e quatro 1's consecutivos.
Mas assim fica difícil para quem começou a aprender agora.

Abraços

-- 
Esta mensagem foi verificada pelo sistema de antiv�rus e
 acredita-se estar livre de perigo.



[obm-l] Re: [obm-l] Re: [obm-l] Re: [obm-l] Re: [obm-l] Re: [obm-l] Questão de Cardinalidade

2018-01-21 Por tôpico Anderson Torres
Em 16 de janeiro de 2018 13:50, Bernardo Freitas Paulo da Costa
<bernardo...@gmail.com> escreveu:
> 2018-01-16 1:10 GMT-02:00 Anderson Torres <torres.anderson...@gmail.com>:
>> Eu na verdade pensei ao contrário:
>>
>> Começamos com o conjunto de todos os subconjuntos de N. Cada conjunto
>> será representado por uma string infinita de zeros e unzes, da
>> seguinte forma: Se o conjunto contiver o natural x, o x-ésimo
>> caractere desta string será 1; caso contrário, será 0.
>>
>> Botando zero-vírgula na frente, obtemos um número real escrito em base
>> 2, contido no intervalo [0,1] (para efeito de completude do argumento,
>> admitiremos strings infinitas de 1zes).
>>
>> Para cada real em [0,1], bastaria escrever na base 2 e criar um
>> conjunto a partir daí, seguindo os passos acima (se o X-esimo dígito é
>> 1, escolhe X, caso contrário, despreza X).
>>
>> Isso prova que existe uma bijeção entre o conjunto das partes de N e o
>> intervalo [0,1].
>
> Acho que tanto a sua demonstração como a do Sávio têm um problema:
>
> 0,0111... = 0.1...
>
> Isso quer dizer que o conjunto {0} e o conjunto {1,2,3,...} são
> enviados no mesmo número real (conhecido como 1/2, ou 0.5 em decimal).
>
> Eu sempre acho muita "forçação de barra" tentar exibir uma bijeção.
> 99% das vezes, é mais esforço do que precisa, sem ganhar muito
> entendimento.  Ou, como neste caso, papa-se uma mosca...  Minha
> sugestão é exibir uma sobrejeção de P(IN) em IR, e depois uma
> sobrejeção de IR em P(IN).  A primeira está garantida, pois basta
> compor a construção do número binário em [0,1] com qualquer sobrejeção
> deste conjunto em R.  Uma sobrejeção simples é mandar 0 e 1 "pra
> qualquer lugar", e depois usar uma bijeção de (0,1) em IR.

Claro que tem a questão das formalizações, mas acho que elas são
trabalho demais para compreensão de menos. Só quis exibir algumas
funções que podem ser o que precisamos.


>
> Deixo para vocês pensarem como fazer para exibir uma sobrejeção de IR
> nas partes de IN.  Dica: IR contém [0,1) e [1,2).

Diretamente? Ainda acho que bijetar toda a reta em um de seus
segmentos uma jogada mais interessante...

>
> Abraços,
> --
> Bernardo Freitas Paulo da Costa
>
> --
> Esta mensagem foi verificada pelo sistema de antivírus e
>  acredita-se estar livre de perigo.
>
>
> =
> Instru�ões para entrar na lista, sair da lista e usar a lista em
> http://www.mat.puc-rio.br/~obmlistas/obm-l.html
> =

-- 
Esta mensagem foi verificada pelo sistema de antiv�rus e
 acredita-se estar livre de perigo.


=
Instru��es para entrar na lista, sair da lista e usar a lista em
http://www.mat.puc-rio.br/~obmlistas/obm-l.html
=


[obm-l] Re: [obm-l] Re: [obm-l] Re: [obm-l] Re: [obm-l] Re: [obm-l] Re: [obm-l] Questão de Cardinalidade

2018-01-16 Por tôpico Bernardo Freitas Paulo da Costa
2018-01-16 14:11 GMT-02:00 Igor Caetano Diniz :
> Fala Bernardo, tudo certo?
> Mas sera que eu conseguiria provar que esses números não seriam uma
> quantidade enumeravel de pontos entre 0 e 1 e, então, como é enumeravel, eu
> consigo pegar uma quantidade enumeravel em P(N) para esses pontos.

Sim, de fato são enumeráveis (é um exercício legal provar isto).  Dá
um pouco mais de trabalho "modificar" as bijeções para corrigir o que
está acontecendo nestes pontos

> Acha que
> seria ruim?

Não digo que seja ruim, só acho que é "trabalho demais" quando você
poderia ir por um caminho mais simples ;-)

Abraços,
-- 
Bernardo Freitas Paulo da Costa

-- 
Esta mensagem foi verificada pelo sistema de antiv�rus e
 acredita-se estar livre de perigo.


=
Instru��es para entrar na lista, sair da lista e usar a lista em
http://www.mat.puc-rio.br/~obmlistas/obm-l.html
=


[obm-l] Re: [obm-l] Re: [obm-l] Re: [obm-l] Re: [obm-l] Re: [obm-l] Questão de Cardinalidade

2018-01-16 Por tôpico Igor Caetano Diniz
Fala Bernardo, tudo certo?
Mas sera que eu conseguiria provar que esses números não seriam uma
quantidade enumeravel de pontos entre 0 e 1 e, então, como é enumeravel, eu
consigo pegar uma quantidade enumeravel em P(N) para esses pontos. Acha que
seria ruim?

Abraço

On Jan 16, 2018 13:59, "Bernardo Freitas Paulo da Costa" <
bernardo...@gmail.com> wrote:

> 2018-01-16 1:10 GMT-02:00 Anderson Torres :
> > Eu na verdade pensei ao contrário:
> >
> > Começamos com o conjunto de todos os subconjuntos de N. Cada conjunto
> > será representado por uma string infinita de zeros e unzes, da
> > seguinte forma: Se o conjunto contiver o natural x, o x-ésimo
> > caractere desta string será 1; caso contrário, será 0.
> >
> > Botando zero-vírgula na frente, obtemos um número real escrito em base
> > 2, contido no intervalo [0,1] (para efeito de completude do argumento,
> > admitiremos strings infinitas de 1zes).
> >
> > Para cada real em [0,1], bastaria escrever na base 2 e criar um
> > conjunto a partir daí, seguindo os passos acima (se o X-esimo dígito é
> > 1, escolhe X, caso contrário, despreza X).
> >
> > Isso prova que existe uma bijeção entre o conjunto das partes de N e o
> > intervalo [0,1].
>
> Acho que tanto a sua demonstração como a do Sávio têm um problema:
>
> 0,0111... = 0.1...
>
> Isso quer dizer que o conjunto {0} e o conjunto {1,2,3,...} são
> enviados no mesmo número real (conhecido como 1/2, ou 0.5 em decimal).
>
> Eu sempre acho muita "forçação de barra" tentar exibir uma bijeção.
> 99% das vezes, é mais esforço do que precisa, sem ganhar muito
> entendimento.  Ou, como neste caso, papa-se uma mosca...  Minha
> sugestão é exibir uma sobrejeção de P(IN) em IR, e depois uma
> sobrejeção de IR em P(IN).  A primeira está garantida, pois basta
> compor a construção do número binário em [0,1] com qualquer sobrejeção
> deste conjunto em R.  Uma sobrejeção simples é mandar 0 e 1 "pra
> qualquer lugar", e depois usar uma bijeção de (0,1) em IR.
>
> Deixo para vocês pensarem como fazer para exibir uma sobrejeção de IR
> nas partes de IN.  Dica: IR contém [0,1) e [1,2).
>
> Abraços,
> --
> Bernardo Freitas Paulo da Costa
>
> --
> Esta mensagem foi verificada pelo sistema de antivírus e
>  acredita-se estar livre de perigo.
>
>
> =
> Instru�ões para entrar na lista, sair da lista e usar a lista em
> http://www.mat.puc-rio.br/~obmlistas/obm-l.html
> =
>

-- 
Esta mensagem foi verificada pelo sistema de antiv�rus e
 acredita-se estar livre de perigo.



[obm-l] Re: [obm-l] Re: [obm-l] Re: [obm-l] Re: [obm-l] Questão de Cardinalidade

2018-01-16 Por tôpico Bernardo Freitas Paulo da Costa
2018-01-16 1:10 GMT-02:00 Anderson Torres :
> Eu na verdade pensei ao contrário:
>
> Começamos com o conjunto de todos os subconjuntos de N. Cada conjunto
> será representado por uma string infinita de zeros e unzes, da
> seguinte forma: Se o conjunto contiver o natural x, o x-ésimo
> caractere desta string será 1; caso contrário, será 0.
>
> Botando zero-vírgula na frente, obtemos um número real escrito em base
> 2, contido no intervalo [0,1] (para efeito de completude do argumento,
> admitiremos strings infinitas de 1zes).
>
> Para cada real em [0,1], bastaria escrever na base 2 e criar um
> conjunto a partir daí, seguindo os passos acima (se o X-esimo dígito é
> 1, escolhe X, caso contrário, despreza X).
>
> Isso prova que existe uma bijeção entre o conjunto das partes de N e o
> intervalo [0,1].

Acho que tanto a sua demonstração como a do Sávio têm um problema:

0,0111... = 0.1...

Isso quer dizer que o conjunto {0} e o conjunto {1,2,3,...} são
enviados no mesmo número real (conhecido como 1/2, ou 0.5 em decimal).

Eu sempre acho muita "forçação de barra" tentar exibir uma bijeção.
99% das vezes, é mais esforço do que precisa, sem ganhar muito
entendimento.  Ou, como neste caso, papa-se uma mosca...  Minha
sugestão é exibir uma sobrejeção de P(IN) em IR, e depois uma
sobrejeção de IR em P(IN).  A primeira está garantida, pois basta
compor a construção do número binário em [0,1] com qualquer sobrejeção
deste conjunto em R.  Uma sobrejeção simples é mandar 0 e 1 "pra
qualquer lugar", e depois usar uma bijeção de (0,1) em IR.

Deixo para vocês pensarem como fazer para exibir uma sobrejeção de IR
nas partes de IN.  Dica: IR contém [0,1) e [1,2).

Abraços,
-- 
Bernardo Freitas Paulo da Costa

-- 
Esta mensagem foi verificada pelo sistema de antiv�rus e
 acredita-se estar livre de perigo.


=
Instru��es para entrar na lista, sair da lista e usar a lista em
http://www.mat.puc-rio.br/~obmlistas/obm-l.html
=


[obm-l] Re: [obm-l] Re: [obm-l] Re: [obm-l] Re: [obm-l] Questão de Cardinalidade

2018-01-16 Por tôpico Igor Caetano Diniz
Uma ideia legal Para provar que (-1,1) tem bijeção com R, seria usar f(x) =
x/(x^2-1) provando que ela eh injetiva e sobrejetiva

On Jan 16, 2018 01:20, "Anderson Torres" <torres.anderson...@gmail.com>
wrote:

> Eu na verdade pensei ao contrário:
>
> Começamos com o conjunto de todos os subconjuntos de N. Cada conjunto
> será representado por uma string infinita de zeros e unzes, da
> seguinte forma: Se o conjunto contiver o natural x, o x-ésimo
> caractere desta string será 1; caso contrário, será 0.
>
> Botando zero-vírgula na frente, obtemos um número real escrito em base
> 2, contido no intervalo [0,1] (para efeito de completude do argumento,
> admitiremos strings infinitas de 1zes).
>
> Para cada real em [0,1], bastaria escrever na base 2 e criar um
> conjunto a partir daí, seguindo os passos acima (se o X-esimo dígito é
> 1, escolhe X, caso contrário, despreza X).
>
> Isso prova que existe uma bijeção entre o conjunto das partes de N e o
> intervalo [0,1].
>
> Agora, provar que [0,1] tem a mesma cardinalidade que R é mais
> chatinho. Dá para pensar geometricamente:
>
> Primeiro, [0,1] tem a mesma cardinalidade de [-1,+1], basta dobrar e
> tirar 1 (f(x)=2x-1).
>
> Agora, como demonstrar que [-1,+1] bijeta com todos os reais? Bem,
> isso não me parece complicado: se pensarmos na inversão de centro zero
> e raio um, o elemento X<1 vai ser levado em 1/X>1. Assim, todo número
> fora de [-1,+1] é bijetado com um dentro de [-1,+1] - podemos
> convencionar que -1,0,+1 vão neles mesmos.
>
> Para sermos mais precisos, o intervalo [0,1] é bijetado em [1,+inf], e
> o intervalo [-1,0] em [-inf,-1]
>
> Agora vem o toque final: acrescente 1 em cada elemento do intervalo
> [-inf,-1], diminua 1 em cada elemento de [1,+inf] e una os resultados.
> Com isso, obtemos uma bijeção de [-inf,-1] união [1,+inf] com toda a
> reta!
>
> E acabou!
> Em 15 de janeiro de 2018 17:11, Igor Caetano Diniz
> <icaetanodi...@gmail.com> escreveu:
> > Olá Sávio,
> > Muito obrigado. Tava pensando em algo parecido mas agora voce esclareceu
> > bastante.
> > Abraços
> >
> > On Jan 15, 2018 16:55, "Sávio Ribas" <savio.ri...@gmail.com> wrote:
> >>
> >> Boa tarde!
> >> A primeira parte servirá para mostrar que a cardinalidade de IR é igual
> à
> >> cardinalidade de [0,1].
> >> Não é difícil mostrar que a reta tem a mesma cardinalidade que, por
> >> exemplo, o intervalo (-1,1) -- basta tomar a bijeção f: (-1,1) -> IR
> dada
> >> por f(x) = tg(pi*x/2).
> >> O passo seguinte seria mostrar que (-1,1) tem a mesma cardinalidade que
> o
> >> intervalo (fechado) [0,1], e para isso vamos tomar a bijeção g: (0,1) ->
> >> (-1,1) dada por g(x) = 2x-1. Mas note que "faltam o pontos 0 e 1" no
> domínio
> >> de g. Vamos acrescentar esses pontos, tomando um conjunto enumerável A =
> >> {a_1, a_2, a_3,...} contido em (0,1) e fazendo o seguinte: Seja B = {0,
> 1,
> >> a_1, a_2, a_3, ...}. A função h: (0,1) -> [0,1] dada por h(x) = x se x
> não
> >> está em A, h(a_1) = 0, h(a_2) = 1, h(a_n) = a_{n-2} se n>2 é uma bijeção
> >> (verifique).
> >> Assim, a função [ h o g^(-1) o f^(-1) ]: IR -> [0,1] é uma bijeção. Daí,
> >> concluímos que IR e [0,1] possuem a mesma cardinalidade.
> >>
> >> Vamos agora mostrar que as cardinalidades de [0,1] e IN são iguais. Seja
> >> 0,b_1b_2b_3... a representação binária de um número em [0,1] com
> infinitas
> >> casas (por exemplo, 1 será representado por 0,1...). Essa escrita
> >> binária dos elementos de [0,1] gera uma bijeção com as partes de IN da
> >> seguinte forma: k perntence a um subconjunto M dos naturais se e
> somente se
> >> b_k = 1 (por exemplo, o vazio corresponde ao 0 = 0,..., IN
> corresponde
> >> ao 1 = 0,... e {2,3,5,7} corresponde a 0,01101010...). Dessa
> forma,
> >> construímos uma bijeção entre P(IN) e [0,1].
> >>
> >> Concluímos que P(IN) e IR possuem mesma cardinalidade, pois ambos estão
> em
> >> bijeção com [0,1].
> >>
> >> Sávio
> >>
> >>
> >> Em 15 de jan de 2018 13:43, "Igor Caetano Diniz" <
> icaetanodi...@gmail.com>
> >> escreveu:
> >>>
> >>> Olá a todos, estou com uma dúvida para provar uma questão(Sem usar
> >>> hipótese do contínuo)
> >>>
> >>> Prove que a cardinalidade do conjunto das partes dos números naturais é
> >>> igual à cardinalidade dos reais, i.e., |P(N)| = |R|
> >>>
> >>>
> >>> 

[obm-l] Re: [obm-l] Re: [obm-l] Re: [obm-l] Questão de Cardinalidade

2018-01-15 Por tôpico Anderson Torres
Eu na verdade pensei ao contrário:

Começamos com o conjunto de todos os subconjuntos de N. Cada conjunto
será representado por uma string infinita de zeros e unzes, da
seguinte forma: Se o conjunto contiver o natural x, o x-ésimo
caractere desta string será 1; caso contrário, será 0.

Botando zero-vírgula na frente, obtemos um número real escrito em base
2, contido no intervalo [0,1] (para efeito de completude do argumento,
admitiremos strings infinitas de 1zes).

Para cada real em [0,1], bastaria escrever na base 2 e criar um
conjunto a partir daí, seguindo os passos acima (se o X-esimo dígito é
1, escolhe X, caso contrário, despreza X).

Isso prova que existe uma bijeção entre o conjunto das partes de N e o
intervalo [0,1].

Agora, provar que [0,1] tem a mesma cardinalidade que R é mais
chatinho. Dá para pensar geometricamente:

Primeiro, [0,1] tem a mesma cardinalidade de [-1,+1], basta dobrar e
tirar 1 (f(x)=2x-1).

Agora, como demonstrar que [-1,+1] bijeta com todos os reais? Bem,
isso não me parece complicado: se pensarmos na inversão de centro zero
e raio um, o elemento X<1 vai ser levado em 1/X>1. Assim, todo número
fora de [-1,+1] é bijetado com um dentro de [-1,+1] - podemos
convencionar que -1,0,+1 vão neles mesmos.

Para sermos mais precisos, o intervalo [0,1] é bijetado em [1,+inf], e
o intervalo [-1,0] em [-inf,-1]

Agora vem o toque final: acrescente 1 em cada elemento do intervalo
[-inf,-1], diminua 1 em cada elemento de [1,+inf] e una os resultados.
Com isso, obtemos uma bijeção de [-inf,-1] união [1,+inf] com toda a
reta!

E acabou!
Em 15 de janeiro de 2018 17:11, Igor Caetano Diniz
<icaetanodi...@gmail.com> escreveu:
> Olá Sávio,
> Muito obrigado. Tava pensando em algo parecido mas agora voce esclareceu
> bastante.
> Abraços
>
> On Jan 15, 2018 16:55, "Sávio Ribas" <savio.ri...@gmail.com> wrote:
>>
>> Boa tarde!
>> A primeira parte servirá para mostrar que a cardinalidade de IR é igual à
>> cardinalidade de [0,1].
>> Não é difícil mostrar que a reta tem a mesma cardinalidade que, por
>> exemplo, o intervalo (-1,1) -- basta tomar a bijeção f: (-1,1) -> IR dada
>> por f(x) = tg(pi*x/2).
>> O passo seguinte seria mostrar que (-1,1) tem a mesma cardinalidade que o
>> intervalo (fechado) [0,1], e para isso vamos tomar a bijeção g: (0,1) ->
>> (-1,1) dada por g(x) = 2x-1. Mas note que "faltam o pontos 0 e 1" no domínio
>> de g. Vamos acrescentar esses pontos, tomando um conjunto enumerável A =
>> {a_1, a_2, a_3,...} contido em (0,1) e fazendo o seguinte: Seja B = {0, 1,
>> a_1, a_2, a_3, ...}. A função h: (0,1) -> [0,1] dada por h(x) = x se x não
>> está em A, h(a_1) = 0, h(a_2) = 1, h(a_n) = a_{n-2} se n>2 é uma bijeção
>> (verifique).
>> Assim, a função [ h o g^(-1) o f^(-1) ]: IR -> [0,1] é uma bijeção. Daí,
>> concluímos que IR e [0,1] possuem a mesma cardinalidade.
>>
>> Vamos agora mostrar que as cardinalidades de [0,1] e IN são iguais. Seja
>> 0,b_1b_2b_3... a representação binária de um número em [0,1] com infinitas
>> casas (por exemplo, 1 será representado por 0,1...). Essa escrita
>> binária dos elementos de [0,1] gera uma bijeção com as partes de IN da
>> seguinte forma: k perntence a um subconjunto M dos naturais se e somente se
>> b_k = 1 (por exemplo, o vazio corresponde ao 0 = 0,..., IN corresponde
>> ao 1 = 0,... e {2,3,5,7} corresponde a 0,01101010...). Dessa forma,
>> construímos uma bijeção entre P(IN) e [0,1].
>>
>> Concluímos que P(IN) e IR possuem mesma cardinalidade, pois ambos estão em
>> bijeção com [0,1].
>>
>> Sávio
>>
>>
>> Em 15 de jan de 2018 13:43, "Igor Caetano Diniz" <icaetanodi...@gmail.com>
>> escreveu:
>>>
>>> Olá a todos, estou com uma dúvida para provar uma questão(Sem usar
>>> hipótese do contínuo)
>>>
>>> Prove que a cardinalidade do conjunto das partes dos números naturais é
>>> igual à cardinalidade dos reais, i.e., |P(N)| = |R|
>>>
>>>
>>> quem puder ajudar, agradeço.
>>>
>>> Abraços
>>>
>>> --
>>> Esta mensagem foi verificada pelo sistema de antivírus e
>>> acredita-se estar livre de perigo.
>>
>>
>> --
>> Esta mensagem foi verificada pelo sistema de antivírus e
>> acredita-se estar livre de perigo.
>
>
> --
> Esta mensagem foi verificada pelo sistema de antivírus e
> acredita-se estar livre de perigo.

-- 
Esta mensagem foi verificada pelo sistema de antiv�rus e
 acredita-se estar livre de perigo.


=
Instru��es para entrar na lista, sair da lista e usar a lista em
http://www.mat.puc-rio.br/~obmlistas/obm-l.html
=


[obm-l] Re: [obm-l] Re: [obm-l] Questão de Cardinalidade

2018-01-15 Por tôpico Igor Caetano Diniz
Olá Sávio,
Muito obrigado. Tava pensando em algo parecido mas agora voce esclareceu
bastante.
Abraços

On Jan 15, 2018 16:55, "Sávio Ribas" <savio.ri...@gmail.com> wrote:

> Boa tarde!
> A primeira parte servirá para mostrar que a cardinalidade de IR é igual à
> cardinalidade de [0,1].
> Não é difícil mostrar que a reta tem a mesma cardinalidade que, por
> exemplo, o intervalo (-1,1) -- basta tomar a bijeção f: (-1,1) -> IR dada
> por f(x) = tg(pi*x/2).
> O passo seguinte seria mostrar que (-1,1) tem a mesma cardinalidade que o
> intervalo (fechado) [0,1], e para isso vamos tomar a bijeção g: (0,1) ->
> (-1,1) dada por g(x) = 2x-1. Mas note que "faltam o pontos 0 e 1" no
> domínio de g. Vamos acrescentar esses pontos, tomando um conjunto
> enumerável A = {a_1, a_2, a_3,...} contido em (0,1) e fazendo o seguinte:
> Seja B = {0, 1, a_1, a_2, a_3, ...}. A função h: (0,1) -> [0,1] dada por
> h(x) = x se x não está em A, h(a_1) = 0, h(a_2) = 1, h(a_n) = a_{n-2} se
> n>2 é uma bijeção (verifique).
> Assim, a função [ h o g^(-1) o f^(-1) ]: IR -> [0,1] é uma bijeção. Daí,
> concluímos que IR e [0,1] possuem a mesma cardinalidade.
>
> Vamos agora mostrar que as cardinalidades de [0,1] e IN são iguais. Seja
> 0,b_1b_2b_3... a representação binária de um número em [0,1] com infinitas
> casas (por exemplo, 1 será representado por 0,1...). Essa escrita
> binária dos elementos de [0,1] gera uma bijeção com as partes de IN da
> seguinte forma: k perntence a um subconjunto M dos naturais se e somente se
> b_k = 1 (por exemplo, o vazio corresponde ao 0 = 0,..., IN corresponde
> ao 1 = 0,... e {2,3,5,7} corresponde a 0,01101010...). Dessa forma,
> construímos uma bijeção entre P(IN) e [0,1].
>
> Concluímos que P(IN) e IR possuem mesma cardinalidade, pois ambos estão em
> bijeção com [0,1].
>
> Sávio
>
>
> Em 15 de jan de 2018 13:43, "Igor Caetano Diniz" <icaetanodi...@gmail.com>
> escreveu:
>
>> Olá a todos, estou com uma dúvida para provar uma questão(Sem usar
>> hipótese do contínuo)
>>
>> Prove que a cardinalidade do conjunto das partes dos números naturais é
>> igual à cardinalidade dos reais, i.e., |P(N)| = |R|
>>
>>
>> quem puder ajudar, agradeço.
>>
>> Abraços
>>
>> --
>> Esta mensagem foi verificada pelo sistema de antivírus e
>> acredita-se estar livre de perigo.
>
>
> --
> Esta mensagem foi verificada pelo sistema de antivírus e
> acredita-se estar livre de perigo.

-- 
Esta mensagem foi verificada pelo sistema de antiv�rus e
 acredita-se estar livre de perigo.



[obm-l] Re: [obm-l] Questão de Cardinalidade

2018-01-15 Por tôpico Sávio Ribas
Boa tarde!
A primeira parte servirá para mostrar que a cardinalidade de IR é igual à
cardinalidade de [0,1].
Não é difícil mostrar que a reta tem a mesma cardinalidade que, por
exemplo, o intervalo (-1,1) -- basta tomar a bijeção f: (-1,1) -> IR dada
por f(x) = tg(pi*x/2).
O passo seguinte seria mostrar que (-1,1) tem a mesma cardinalidade que o
intervalo (fechado) [0,1], e para isso vamos tomar a bijeção g: (0,1) ->
(-1,1) dada por g(x) = 2x-1. Mas note que "faltam o pontos 0 e 1" no
domínio de g. Vamos acrescentar esses pontos, tomando um conjunto
enumerável A = {a_1, a_2, a_3,...} contido em (0,1) e fazendo o seguinte:
Seja B = {0, 1, a_1, a_2, a_3, ...}. A função h: (0,1) -> [0,1] dada por
h(x) = x se x não está em A, h(a_1) = 0, h(a_2) = 1, h(a_n) = a_{n-2} se
n>2 é uma bijeção (verifique).
Assim, a função [ h o g^(-1) o f^(-1) ]: IR -> [0,1] é uma bijeção. Daí,
concluímos que IR e [0,1] possuem a mesma cardinalidade.

Vamos agora mostrar que as cardinalidades de [0,1] e IN são iguais. Seja
0,b_1b_2b_3... a representação binária de um número em [0,1] com infinitas
casas (por exemplo, 1 será representado por 0,1...). Essa escrita
binária dos elementos de [0,1] gera uma bijeção com as partes de IN da
seguinte forma: k perntence a um subconjunto M dos naturais se e somente se
b_k = 1 (por exemplo, o vazio corresponde ao 0 = 0,..., IN corresponde
ao 1 = 0,... e {2,3,5,7} corresponde a 0,01101010...). Dessa forma,
construímos uma bijeção entre P(IN) e [0,1].

Concluímos que P(IN) e IR possuem mesma cardinalidade, pois ambos estão em
bijeção com [0,1].

Sávio


Em 15 de jan de 2018 13:43, "Igor Caetano Diniz" <icaetanodi...@gmail.com>
escreveu:

> Olá a todos, estou com uma dúvida para provar uma questão(Sem usar
> hipótese do contínuo)
>
> Prove que a cardinalidade do conjunto das partes dos números naturais é
> igual à cardinalidade dos reais, i.e., |P(N)| = |R|
>
>
> quem puder ajudar, agradeço.
>
> Abraços
>
> --
> Esta mensagem foi verificada pelo sistema de antivírus e
> acredita-se estar livre de perigo.

-- 
Esta mensagem foi verificada pelo sistema de antiv�rus e
 acredita-se estar livre de perigo.



[obm-l] Questão de Cardinalidade

2018-01-15 Por tôpico Igor Caetano Diniz
Olá a todos, estou com uma dúvida para provar uma questão(Sem usar hipótese
do contínuo)

Prove que a cardinalidade do conjunto das partes dos números naturais é
igual à cardinalidade dos reais, i.e., |P(N)| = |R|


quem puder ajudar, agradeço.

Abraços

-- 
Esta mensagem foi verificada pelo sistema de antiv�rus e
 acredita-se estar livre de perigo.



[obm-l] Re: [obm-l] Enc: Questão da OBM

2018-01-03 Por tôpico Anderson Torres
Tem uma resposta do pessoal do MathLinks, que usa um pouco de Teoria
dos Grupos. A ideia é que a pode ser escrito na forma (t+t^(-1)), e
daí este t acaba sendo uma raiz nona de 1.

https://artofproblemsolving.com/community/c6h1556461p9495218

Em 2 de janeiro de 2018 21:06, marcone augusto araújo borges
<marconeborge...@hotmail.com> escreveu:
>
>
>
> 
> De: marcone augusto araújo borges <marconeborge...@hotmail.com>
> Enviado: segunda-feira, 1 de janeiro de 2018 15:06
> Para: obm-l@mat.puc-rio.br
> Assunto: Questão da OBM
>
>
> Seja a inteiro positivo e p um divisor primo de a^3 -3a +1 com p diferente
> de 3.
>
> Prove que p é da forma 9k + 1 ou 9k - 1, sendo k inteiro.
>
> Como resolver?
>
>
> --
> Esta mensagem foi verificada pelo sistema de antivírus e
> acredita-se estar livre de perigo.

-- 
Esta mensagem foi verificada pelo sistema de antiv�rus e
 acredita-se estar livre de perigo.


=
Instru��es para entrar na lista, sair da lista e usar a lista em
http://www.mat.puc-rio.br/~obmlistas/obm-l.html
=


[obm-l] Enc: Questão da OBM

2018-01-02 Por tôpico marcone augusto araújo borges




De: marcone augusto araújo borges <marconeborge...@hotmail.com>
Enviado: segunda-feira, 1 de janeiro de 2018 15:06
Para: obm-l@mat.puc-rio.br
Assunto: Questão da OBM


Seja a inteiro positivo e p um divisor primo de a^3 -3a +1 com p diferente de 3.

Prove que p é da forma 9k + 1 ou 9k - 1, sendo k inteiro.

Como resolver?

-- 
Esta mensagem foi verificada pelo sistema de antivírus e
 acredita-se estar livre de perigo.



[obm-l] Questão da OBM

2018-01-01 Por tôpico marcone augusto araújo borges
Seja a inteiro positivo e p um divisor primo de a^3 -3a +1 com p diferente de 3.

Prove que p é da forma 9k + 1 ou 9k - 1, sendo k inteiro.

Como resolver?

-- 
Esta mensagem foi verificada pelo sistema de antivírus e
 acredita-se estar livre de perigo.



[obm-l] Re: [obm-l] Re: [obm-l] Questão

2017-08-28 Por tôpico Pedro José
Bom dia!

Daniel,
eu já me sinto gratificado quando consigo resolver algo. Não sou
matemático, sou um pitaqueiro, com alto grau curiosidade e matemática é uma
das minhas curiosidades preferidas.
O que mais me fascina, é que sou totalmente crente em que um modelo
matemático formulado com estrutura, se não tem aplicação prática é porque
não enxergamos onde podemos usá-lo no momento e não por que não tenha
utilização, como alguns leigos (como eu, somente no fato de ser leigo)
 apregoam.
A matemática é a mais veloz das ciências.
O Gabriel falou em ordem, fez uma observação e depois se corrigiu.
O difícil aqui é saber o nível de conhecimento das pessoas, por isso que
não falei em ordem, mas se você não souber o que é ordem, pesquise.
Pesquise também o que seria uma raiz primitiva, caso não conheça o
conceito, pois 10 é raiz primitiva módulo 23.

Saudações,
PJMS

Em 25 de agosto de 2017 16:51, Gabriel Tostes  escreveu:

> Confundi, eh 22 msm. :D
>
> On Aug 25, 2017, at 12:28 AM, Daniel da Silva <
> danielrochadasi...@icloud.com> wrote:
>
> Obrigado Pedro.
>
> Daniel Rocha da Silva
>
> Em 23 de ago de 2017, às 19:31, Pedro José 
> escreveu:
>
> Boa noite!
>
> O difícil é achar o n.
>
> Como o menor inteiro positivo que atende 10^a = 1 mod23 é a=22
>
> E como 10^3 = 11 mod23.
>
> Temos que K + 1 = 3 +22*m com m natural
> então k = 2 + 22*m.
>
> e n/2 = [10^(k+1) -11]/23 ==> n=2*[10^(k+1)-11]/23.
>
> Portanto as soluções serão (2+ 22*m; 2*[10^(3+22*m)-11]/23; com
> m= 0,1, 2, 3, 4
>
> Então há uma infinidade de soluções. você achou a relativa
> a m=0.
>
> ou seja, k= 2 e n = 2*[10^3-11]/23=2*43=86
>
> para m =1; k= 24 e n= 869,575.217.391.304.347.826.086
>
> Salvo engano para n pois fiz na marra.
>
> Saudações,
> PJMS
>
> Em 23 de agosto de 2017 17:19, Daniel da Silva <
> danielrochadasi...@icloud.com> escreveu:
>
>> Boa tarde,
>>
>> Como saber quantos valores inteiros
>> de N e K satisfazem a seguinte equação:
>>
>> 10^(K+1)=11+23N/2
>>
>> Encontrei uma solução (N=86, K=2), mas como saber se
>> é única?
>>
>> Obrigado,
>> Daniel Rocha da Silva
>> --
>> Esta mensagem foi verificada pelo sistema de antivírus e
>> Â acredita-se estar livre de perigo.
>>
>>
>> =
>> Instruções para entrar na lista, sair da lista e usar a lista em
>> http://www.mat.puc-rio.br/~obmlistas/obm-l.html
>> =
>>
>
>
> --
> Esta mensagem foi verificada pelo sistema de antivírus e
> acredita-se estar livre de perigo.
>
>
> --
> Esta mensagem foi verificada pelo sistema de antivírus e
> acredita-se estar livre de perigo.
>
>
> --
> Esta mensagem foi verificada pelo sistema de antivírus e
> acredita-se estar livre de perigo.
>

-- 
Esta mensagem foi verificada pelo sistema de antiv�rus e
 acredita-se estar livre de perigo.



Re: [obm-l] Re: [obm-l] Questão

2017-08-25 Por tôpico Gabriel Tostes
Faltou so uma coisa, a ordem de 10 mod 23 é 11 nao 22. Entao o k= 2+11k 


> On Aug 25, 2017, at 12:28 AM, Daniel da Silva  
> wrote:
> 
> Obrigado Pedro.
> 
> Daniel Rocha da Silva
> 
> Em 23 de ago de 2017, às 19:31, Pedro José  escreveu:
> 
>> Boa noite!
>> 
>> O difícil é achar o n.
>> 
>> Como o menor inteiro positivo que atende 10^a = 1 mod23 é a=22
>> 
>> E como 10^3 = 11 mod23.
>> 
>> Temos que K + 1 = 3 +22*m com m natural
>> então k = 2 + 22*m.
>> 
>> e n/2 = [10^(k+1) -11]/23 ==> n=2*[10^(k+1)-11]/23.
>> 
>> Portanto as soluções serão (2+ 22*m; 2*[10^(3+22*m)-11]/23; com m= 
>> 0,1, 2, 3, 4
>> 
>> Então há uma infinidade de soluções. você achou a relativa a 
>> m=0.
>> 
>> ou seja, k= 2 e n = 2*[10^3-11]/23=2*43=86
>> 
>> para m =1; k= 24 e n= 869,575.217.391.304.347.826.086
>> 
>> Salvo engano para n pois fiz na marra.
>> 
>> Saudações,
>> PJMS
>> 
>> Em 23 de agosto de 2017 17:19, Daniel da Silva 
>>  escreveu:
>>> Boa tarde,
>>> 
>>> Como saber quantos valores inteiros
>>> de N e K satisfazem a seguinte equação:
>>> 
>>> 10^(K+1)=11+23N/2
>>> 
>>> Encontrei uma solução (N=86, K=2), mas como saber se é 
>>> única?
>>> 
>>> Obrigado,
>>> Daniel Rocha da Silva
>>> --
>>> Esta mensagem foi verificada pelo sistema de antivírus e
>>> Â acredita-se estar livre de perigo.
>>> 
>>> 
>>> =
>>> Instruções para entrar na lista, sair da lista e usar a lista em
>>> http://www.mat.puc-rio.br/~obmlistas/obm-l.html
>>> =
>> 
>> 
>> -- 
>> Esta mensagem foi verificada pelo sistema de antivírus e 
>> acredita-se estar livre de perigo.
> 
> -- 
> Esta mensagem foi verificada pelo sistema de antivírus e 
> acredita-se estar livre de perigo.

-- 
Esta mensagem foi verificada pelo sistema de antiv�rus e
 acredita-se estar livre de perigo.



Re: [obm-l] Re: [obm-l] Questão

2017-08-25 Por tôpico Gabriel Tostes
Confundi, eh 22 msm. :D

> On Aug 25, 2017, at 12:28 AM, Daniel da Silva  
> wrote:
> 
> Obrigado Pedro.
> 
> Daniel Rocha da Silva
> 
> Em 23 de ago de 2017, às 19:31, Pedro José  escreveu:
> 
>> Boa noite!
>> 
>> O difícil é achar o n.
>> 
>> Como o menor inteiro positivo que atende 10^a = 1 mod23 é a=22
>> 
>> E como 10^3 = 11 mod23.
>> 
>> Temos que K + 1 = 3 +22*m com m natural
>> então k = 2 + 22*m.
>> 
>> e n/2 = [10^(k+1) -11]/23 ==> n=2*[10^(k+1)-11]/23.
>> 
>> Portanto as soluções serão (2+ 22*m; 2*[10^(3+22*m)-11]/23; com m= 
>> 0,1, 2, 3, 4
>> 
>> Então há uma infinidade de soluções. você achou a relativa a 
>> m=0.
>> 
>> ou seja, k= 2 e n = 2*[10^3-11]/23=2*43=86
>> 
>> para m =1; k= 24 e n= 869,575.217.391.304.347.826.086
>> 
>> Salvo engano para n pois fiz na marra.
>> 
>> Saudações,
>> PJMS
>> 
>> Em 23 de agosto de 2017 17:19, Daniel da Silva 
>>  escreveu:
>>> Boa tarde,
>>> 
>>> Como saber quantos valores inteiros
>>> de N e K satisfazem a seguinte equação:
>>> 
>>> 10^(K+1)=11+23N/2
>>> 
>>> Encontrei uma solução (N=86, K=2), mas como saber se é 
>>> única?
>>> 
>>> Obrigado,
>>> Daniel Rocha da Silva
>>> --
>>> Esta mensagem foi verificada pelo sistema de antivírus e
>>> Â acredita-se estar livre de perigo.
>>> 
>>> 
>>> =
>>> Instruções para entrar na lista, sair da lista e usar a lista em
>>> http://www.mat.puc-rio.br/~obmlistas/obm-l.html
>>> =
>> 
>> 
>> -- 
>> Esta mensagem foi verificada pelo sistema de antivírus e 
>> acredita-se estar livre de perigo.
> 
> -- 
> Esta mensagem foi verificada pelo sistema de antivírus e 
> acredita-se estar livre de perigo.

-- 
Esta mensagem foi verificada pelo sistema de antiv�rus e
 acredita-se estar livre de perigo.



Re: [obm-l] Re: [obm-l] Questão

2017-08-24 Por tôpico Daniel da Silva
Obrigado Pedro.

Daniel Rocha da Silva

> Em 23 de ago de 2017, às 19:31, Pedro José  escreveu:
> 
> Boa noite!
> 
> O difícil é achar o n.
> 
> Como o menor inteiro positivo que atende 10^a = 1 mod23 é a=22
> 
> E como 10^3 = 11 mod23.
> 
> Temos que K + 1 = 3 +22*m com m natural
> então k = 2 + 22*m.
> 
> e n/2 = [10^(k+1) -11]/23 ==> n=2*[10^(k+1)-11]/23.
> 
> Portanto as soluções serão (2+ 22*m; 2*[10^(3+22*m)-11]/23; com m= 0,1, 2, 
> 3, 4
> 
> Então há uma infinidade de soluções. você achou a relativa a m=0.
> 
> ou seja, k= 2 e n = 2*[10^3-11]/23=2*43=86
> 
> para m =1; k= 24 e n= 869,575.217.391.304.347.826.086
> 
> Salvo engano para n pois fiz na marra.
> 
> Saudações,
> PJMS
> 
> Em 23 de agosto de 2017 17:19, Daniel da Silva 
>  escreveu:
>> Boa tarde,
>> 
>> Como saber quantos valores inteiros
>> de N e K satisfazem a seguinte equação:
>> 
>> 10^(K+1)=11+23N/2
>> 
>> Encontrei uma solução (N=86, K=2), mas como saber se é única?
>> 
>> Obrigado,
>> Daniel Rocha da Silva
>> --
>> Esta mensagem foi verificada pelo sistema de antivírus e
>> Â acredita-se estar livre de perigo.
>> 
>> 
>> =
>> Instruções para entrar na lista, sair da lista e usar a lista em
>> http://www.mat.puc-rio.br/~obmlistas/obm-l.html
>> =
> 
> 
> -- 
> Esta mensagem foi verificada pelo sistema de antivírus e 
> acredita-se estar livre de perigo.

-- 
Esta mensagem foi verificada pelo sistema de antiv�rus e
 acredita-se estar livre de perigo.



[obm-l] Re: [obm-l] Questão

2017-08-23 Por tôpico Pedro José
Boa noite!

O difícil é achar o n.

Como o menor inteiro positivo que atende 10^a = 1 mod23 é a=22

E como 10^3 = 11 mod23.

Temos que K + 1 = 3 +22*m com m natural
então k = 2 + 22*m.

e n/2 = [10^(k+1) -11]/23 ==> n=2*[10^(k+1)-11]/23.

Portanto as soluções serão (2+ 22*m; 2*[10^(3+22*m)-11]/23; com m= 0,1, 2,
3, 4

Então há uma infinidade de soluções. você achou a relativa a m=0.

ou seja, k= 2 e n = 2*[10^3-11]/23=2*43=86

para m =1; k= 24 e n= 869,575.217.391.304.347.826.086

Salvo engano para n pois fiz na marra.

Saudações,
PJMS

Em 23 de agosto de 2017 17:19, Daniel da Silva <
danielrochadasi...@icloud.com> escreveu:

> Boa tarde,
>
> Como saber quantos valores inteiros
> de N e K satisfazem a seguinte equação:
>
> 10^(K+1)=11+23N/2
>
> Encontrei uma solução (N=86, K=2), mas como saber se é única?
>
> Obrigado,
> Daniel Rocha da Silva
> --
> Esta mensagem foi verificada pelo sistema de antivírus e
>  acredita-se estar livre de perigo.
>
>
> =
> Instruções para entrar na lista, sair da lista e usar a lista em
> http://www.mat.puc-rio.br/~obmlistas/obm-l.html
> =
>

-- 
Esta mensagem foi verificada pelo sistema de antiv�rus e
 acredita-se estar livre de perigo.



[obm-l] Questão

2017-08-23 Por tôpico Daniel da Silva
Boa tarde,

Como saber quantos valores inteiros
de N e K satisfazem a seguinte equação:

10^(K+1)=11+23N/2

Encontrei uma solução (N=86, K=2), mas como saber se é única?

Obrigado,
Daniel Rocha da Silva
-- 
Esta mensagem foi verificada pelo sistema de antiv�rus e
 acredita-se estar livre de perigo.


=
Instru��es para entrar na lista, sair da lista e usar a lista em
http://www.mat.puc-rio.br/~obmlistas/obm-l.html
=


[obm-l] Re: [obm-l]Re: [obm-l] Re: [obm-l] Re: [obm-l] Questão de teoria numérica

2017-08-01 Por tôpico Pedro Cardoso
Realmente. Se isso serve de desculpa eu escrevi isso assim que acordei.
O que eu quis dizer é que não existem múltiplos de 2017 que terminem em 0 e 
que, ao serem divididos por 10, deixam de ser múltiplos de 2017. Para isso 
existir, 2017 teria que ter um número de fatores 2 diferente do número de 
fatores 5, mostrar porque isso não é difícil: supondo que k tem x fatores 2 e y 
fatores 5 se x>y temos que 5k/10 tem x-1 fatores 2 e y fatores 5, e portanto, 
não é divisível por k, para x escreveu:
>
> Obrigado! Era exatamente isso que a questão anterior sugeria, usar o 
> princípio da casa dos pombos.
> Uma coisa que percebi na sua dsmonstração é que o número encontrado 
> terminaria em 0s, mas como nenhum multiplo de 2017 também é multiplo de 10 
> (2017 é primo) então também existe um multiplo de 2017 com apenas 1s!

A conclusão está certa (existe um múltiplo de 2017 só com 1s), mas a
justificativa está errada: 20170 é múltiplo de 2017 e de 10.  Tem a
ver com primalidade, mas não é bem o que você escreveu.

Abraços,
-- 
Bernardo Freitas Paulo da Costa

-- 
Esta mensagem foi verificada pelo sistema de antiv�rus e
 acredita-se estar livre de perigo.


=
Instru��es para entrar na lista, sair da lista e usar a lista em
http://www.mat.puc-rio.br/~obmlistas/obm-l.html
=


-- 
Esta mensagem foi verificada pelo sistema de antiv�rus e
 acredita-se estar livre de perigo.



[obm-l] Re: [obm-l] Re: [obm-l] Re: [obm-l] Questão de teoria numérica

2017-08-01 Por tôpico Bernardo Freitas Paulo da Costa
Em 01/08/2017 08:14, "Pedro Cardoso" <mr.pedrocard...@gmail.com> escreveu:
>
> Obrigado! Era exatamente isso que a questão anterior sugeria, usar o 
> princípio da casa dos pombos.
> Uma coisa que percebi na sua dsmonstração é que o número encontrado 
> terminaria em 0s, mas como nenhum multiplo de 2017 também é multiplo de 10 
> (2017 é primo) então também existe um multiplo de 2017 com apenas 1s!

A conclusão está certa (existe um múltiplo de 2017 só com 1s), mas a
justificativa está errada: 20170 é múltiplo de 2017 e de 10.  Tem a
ver com primalidade, mas não é bem o que você escreveu.

Abraços,
-- 
Bernardo Freitas Paulo da Costa

-- 
Esta mensagem foi verificada pelo sistema de antiv�rus e
 acredita-se estar livre de perigo.


=
Instru��es para entrar na lista, sair da lista e usar a lista em
http://www.mat.puc-rio.br/~obmlistas/obm-l.html
=


[obm-l] Re: [obm-l] Re: [obm-l] Questão de teoria numérica

2017-08-01 Por tôpico Pedro Cardoso
Obrigado! Era exatamente isso que a questão anterior sugeria, usar o
princípio da casa dos pombos.
Uma coisa que percebi na sua dsmonstração é que o número encontrado
terminaria em 0s, mas como nenhum multiplo de 2017 também é multiplo de 10
(2017 é primo) então também existe um multiplo de 2017 com apenas 1s!

Obrigado.
Pedro Cardoso

Em 31 de jul de 2017 23:31, "Adilson Francisco da Silva" <
adilson...@gmail.com> escreveu:

> Salve!
>
> Construa uma sequência com 2018 números naturais da seguinte forma:
> 1
> 11
> 111
> 
> .
> .
> .
> 111...1 (2018 dígitos 1).
>
> Pelo princípio da casa dos pombos existe ao menos dois desses números que
> deixam o mesmo resto na divisão por 2017.
>
> Use o fato de que se dois números deixar o mesmo resto na divisão por um
> certo número d, então a diferença entre eles é divisível por d.
>
> Assim pegue os dois termos da sequência que deixa o mesmo resto é faça
>
> 11...111 - 11...1 = 111...100...0
>
> Que é divisível por 2017.
>
> Abraços
>
>
> Em 31 de jul de 2017 10:38 PM, "Pedro Cardoso" <mr.pedrocard...@gmail.com>
> escreveu:
>
> Segue uma questão de teoria numérica da Olimpíada SESI de Matemática (AM):
>
> Mostre que existe um múltiplo de 2017 formado apenas pelos dígitos 0 e 1
> (em base 10).
>
>
>
> Na olimpíada, a questão anterior sugere uma maneira de resolver, porém,
> estou interessado em outras demonstrações também.
>
> Se ninguém conseguir achar uma prova, mando a outra questão mais tarde.
>
> Att.
> Pedro Cardoso
>
>
>
> --
> Esta mensagem foi verificada pelo sistema de antivírus e
> acredita-se estar livre de perigo.
>
>
>
> --
> Esta mensagem foi verificada pelo sistema de antivírus e
> acredita-se estar livre de perigo.

-- 
Esta mensagem foi verificada pelo sistema de antiv�rus e
 acredita-se estar livre de perigo.



[obm-l] Re: [obm-l] Questão de teoria numérica

2017-07-31 Por tôpico Adilson Francisco da Silva
Salve!

Construa uma sequência com 2018 números naturais da seguinte forma:
1
11
111

.
.
.
111...1 (2018 dígitos 1).

Pelo princípio da casa dos pombos existe ao menos dois desses números que
deixam o mesmo resto na divisão por 2017.

Use o fato de que se dois números deixar o mesmo resto na divisão por um
certo número d, então a diferença entre eles é divisível por d.

Assim pegue os dois termos da sequência que deixa o mesmo resto é faça

11...111 - 11...1 = 111...100...0

Que é divisível por 2017.

Abraços


Em 31 de jul de 2017 10:38 PM, "Pedro Cardoso" <mr.pedrocard...@gmail.com>
escreveu:

Segue uma questão de teoria numérica da Olimpíada SESI de Matemática (AM):

Mostre que existe um múltiplo de 2017 formado apenas pelos dígitos 0 e 1
(em base 10).



Na olimpíada, a questão anterior sugere uma maneira de resolver, porém,
estou interessado em outras demonstrações também.

Se ninguém conseguir achar uma prova, mando a outra questão mais tarde.

Att.
Pedro Cardoso



-- 
Esta mensagem foi verificada pelo sistema de antivírus e
acredita-se estar livre de perigo.

-- 
Esta mensagem foi verificada pelo sistema de antiv�rus e
 acredita-se estar livre de perigo.



[obm-l] Questão de teoria numérica

2017-07-31 Por tôpico Pedro Cardoso
Segue uma questão de teoria numérica da Olimpíada SESI de Matemática (AM):

Mostre que existe um múltiplo de 2017 formado apenas pelos dígitos 0 e 1 (em 
base 10).

Na olimpíada, a questão anterior sugere uma maneira de resolver, porém, estou 
interessado em outras demonstrações também. 
Se ninguém conseguir achar uma prova, mando a outra questão mais tarde.
Att.
Pedro Cardoso


-- 
Esta mensagem foi verificada pelo sistema de antiv�rus e
 acredita-se estar livre de perigo.



[obm-l] Questão da IMO.

2017-05-01 Por tôpico heller_h
Olá a todos, estou precisando de ajuda na questão a seguir:(IMO-1994) Encontre todos os pares ordenados (m,n) onde m e n são inteiros positivos tais que (n^3+1)/(mn-1) é um inteiro.Grato desde já,Henrique Heller.--
Esta mensagem foi verificada pelo sistema de antivírus e 
 acredita-se estar livre de perigo.




[obm-l] Re: [obm-l] Questão de um vestibular do Acre

2016-12-23 Por tôpico Pacini Bores
 

Oi Wanderlei, 

Realmente, acredito que falte o ângulo theta, já que ele pede para usar
sqrt(2)=1,4. 

Na verdade o comprimento da maca, para tocar os extremos nas paredes dos
corredores, tem sua limitação dada por 

(p^(2/3)+q^(2/3)^(3/2) se imaginarmos a largura da maca desprezível. 

Abraços 

Pacini 

Em 21/12/2016 16:50, Vanderlei Nemitz escreveu: 

> Boa tarde! 
> Tentei resolver uma questão de um vestibular do Acre, mas parece que faltam 
> informações, que talvez seja necessário supor. 
> Como acho que não posso anexar um arquivo aqui, deixo um link que acessa a 
> prova. É a questão 32, de geometria plana. 
> 
> http://www.strixeducacao.com.br/vs-arquivos/HtmlEditor/file/PROVAS%20APLICADAS/Uninorte_2016_2_Tipo1.pdf
>  [1] 
> 
> Muito obrigado! 
> 
> Vanderlei 
> -- 
> Esta mensagem foi verificada pelo sistema de antivrus e 
> acredita-se estar livre de perigo.
 

Links:
--
[1]
http://www.strixeducacao.com.br/vs-arquivos/HtmlEditor/file/PROVAS%20APLICADAS/Uninorte_2016_2_Tipo1.pdf
-- 
Esta mensagem foi verificada pelo sistema de antiv�rus e
 acredita-se estar livre de perigo.



[obm-l] Questão de um vestibular do Acre

2016-12-21 Por tôpico Vanderlei Nemitz
Boa tarde!
Tentei resolver uma questão de um vestibular do Acre, mas parece que faltam
informações, que talvez seja necessário supor.
Como acho que não posso anexar um arquivo aqui, deixo um link que acessa a
prova. É a questão *32*, de geometria plana.

http://www.strixeducacao.com.br/vs-arquivos/HtmlEditor/file/PROVAS%20APLICADAS/Uninorte_2016_2_Tipo1.pdf

Muito obrigado!

Vanderlei

-- 
Esta mensagem foi verificada pelo sistema de antiv�rus e
 acredita-se estar livre de perigo.



[obm-l] Re: [obm-l] Re: [obm-l] Questão de Física

2016-10-16 Por tôpico Luiz Antonio Rodrigues
Olá, Pacini!
Muito obrigado!
Um abraço!
Luiz
On Oct 16, 2016 10:38 AM, "Pacini Bores" <pacini.bo...@globo.com> wrote:

>
>
>
> Oi Luiz,
>
> o T para pequenas oscilações , T = 2.pi.sqrt(L/g) e com T´=5T=
> 2.pi.sqrt(L/g´), onde g´= (P-q.E)/m.
>
> Logo teremos : (T^2).g = ((T´)^2).g´ ou seja g=25.g´ou g = 25(P-q.E)/m e
> fazendo as contas, encontramos
>
> E = 240N/C.
>
> Abraços
>
> Pacini
>
> Em 15/10/2016 13:49, Luiz Antonio Rodrigues escreveu:
>
> Olá, pessoal! Peço desculpas por postar uma questão de Física, mas preciso
> de ajuda... Já tentei resolvê-la muitas vezes, sem sucesso. Não conheço um
> bom fórum de Física. Desde já agradeço qualquer ajuda. A questão é a
> seguinte:
>
>
> Um pequeno pêndulo simples é posto a oscilar entre duas superfícies
> metálicas planas, quadradas, muito grandes, paralelas e inicialmente
> neutras, apresentando um período T. O pêndulo simples é constituído por
> uma esfera metálica de massa 3,0x10– 4kg, eletrizada com carga de 12μC,
> e um fio isolante de massa desprezível e de comprimento 100cm. Nesse
> local, a aceleração da gravidade vale 10m/s2. A seguir, um dispositivo
> eletriza as placas metálicas, produzindo um campo elétrico uniforme e
> constante orientado para cima. Como as placas metálicas são muito
> grandes, toda a região de oscilação do pêndulo é abrangida pelo campo
> elétrico uniforme, fazendo com que o pêndulo passe a oscilar com um
> período 5T. Nessas condições, a intensidade do campo elétrico uniforme
> e constante estabelecido entre as placas metálicas vale
>
> (A) 125 N/C (B) 150 N/C (C) 200 N/C (D) 240 N/C
>
>
>
> Um abraço!
>
> Luiz
>
> --
> Esta mensagem foi verificada pelo sistema de antivrus e
> acredita-se estar livre de perigo.
>
>
> --
> Esta mensagem foi verificada pelo sistema de antivírus e
> acredita-se estar livre de perigo.
>

-- 
Esta mensagem foi verificada pelo sistema de antiv�rus e
 acredita-se estar livre de perigo.



[obm-l] Re: [obm-l] Questão de Física

2016-10-16 Por tôpico Pacini Bores
 

Oi Luiz, 

o T para pequenas oscilações , T = 2.pi.sqrt(L/g) e com T´=5T=
2.pi.sqrt(L/g´), onde g´= (P-q.E)/m. 

Logo teremos : (T^2).g = ((T´)^2).g´ ou seja g=25.g´ou g = 25(P-q.E)/m e
fazendo as contas, encontramos 

E = 240N/C. 

Abraços 

Pacini 

Em 15/10/2016 13:49, Luiz Antonio Rodrigues escreveu: 

> Olá, pessoal! Peço desculpas por postar uma questão de Física, mas preciso de 
> ajuda... Já tentei resolvê-la muitas vezes, sem sucesso. Não conheço um bom 
> fórum de Física. Desde já agradeço qualquer ajuda. A questão é a seguinte: 
> 
> Um pequeno pêndulo simples é posto a oscilar entre duas superfícies 
> metálicas planas, quadradas, muito grandes, paralelas e inicialmente 
> neutras, apresentando um período T. O pêndulo simples é constituído por 
> uma esfera metálica de massa 3,0x10- 4kg, eletrizada com carga de 12μC, e um 
> fio isolante de massa desprezível e de comprimento 100cm. Nesse local, a 
> aceleração da gravidade vale 10m/s2. A seguir, um dispositivo eletriza as 
> placas metálicas, produzindo um campo elétrico uniforme e constante 
> orientado para cima. Como as placas metálicas são muito grandes, toda a 
> região de oscilação do pêndulo é abrangida pelo campo elétrico 
> uniforme, fazendo com que o pêndulo passe a oscilar com um período 5T. 
> Nessas condições, a intensidade do campo elétrico uniforme e constante 
> estabelecido entre as placas metálicas vale 
> 
> (A) 125 N/C (B) 150 N/C (C) 200 N/C (D) 240 N/C 
> 
> Um abraço! 
> 
> Luiz 
> -- 
> Esta mensagem foi verificada pelo sistema de antivrus e 
> acredita-se estar livre de perigo.
 
-- 
Esta mensagem foi verificada pelo sistema de antiv�rus e
 acredita-se estar livre de perigo.



Re: [obm-l] Questão de Física

2016-10-15 Por tôpico regis barros
Olá LuizO segue fórum vc encontra no yahoo grupos.Física Básica é o nome do 
fórum.
Regis 

Em Sábado, 15 de Outubro de 2016 14:24, Luiz Antonio Rodrigues 
<rodrigue...@gmail.com> escreveu:
 

 Olá, pessoal! Peço desculpas por postar uma questão de Física, mas preciso de 
ajuda... Já tentei resolvê-la muitas vezes, sem sucesso. Não conheço um bom 
fórum de Física. Desde já agradeço qualquer ajuda. A questão é a seguinte: 
 Um pequeno pêndulo simples é posto a oscilar entre duassuperfícies 
metálicas planas, quadradas, muito grandes,paralelas e inicialmente neutras, 
apresentando um período T. O pêndulo simples é constituído por uma 
esferametálica de massa 3,0x10– 4kg, eletrizada com carga de12μC, e um fio 
isolante de massa desprezível e decomprimento 100cm. Nesse local, a 
aceleração dagravidade vale 10m/s2. A seguir, um dispositivo eletriza 
asplacas metálicas, produzindo um campo elétrico uniforme econstante 
orientado para cima. Como as placas metálicassão muito grandes, toda a 
região de oscilação do pênduloé abrangida pelo campo elétrico uniforme, 
fazendo comque o pêndulo passe a oscilar com um período 5T. 
Nessascondições, a intensidade do campo elétrico uniforme econstante 
estabelecido entre as placas metálicas vale (A) 125 N/C(B) 150 N/C(C) 200 
N/C(D) 240 N/C 
Um abraço!Luiz
--
Esta mensagem foi verificada pelo sistema de antiv�us e 
 acredita-se estar livre de perigo.

   
-- 
Esta mensagem foi verificada pelo sistema de antiv�rus e
 acredita-se estar livre de perigo.



[obm-l] Questão de Física

2016-10-15 Por tôpico Luiz Antonio Rodrigues
Olá, pessoal! Peço desculpas por postar uma questão de Física, mas preciso
de ajuda... Já tentei resolvê-la muitas vezes, sem sucesso. Não conheço um
bom fórum de Física. Desde já agradeço qualquer ajuda. A questão é a
seguinte:

Um pequeno pêndulo simples é posto a oscilar entre duas superfícies
metálicas planas, quadradas, muito grandes, paralelas e inicialmente
neutras, apresentando um período T. O pêndulo simples é constituído por
uma esfera metálica de massa 3,0x10– 4kg, eletrizada com carga de 12μC, e
um fio isolante de massa desprezível e de comprimento 100cm. Nesse local,
a aceleração da gravidade vale 10m/s2. A seguir, um dispositivo eletriza
as placas metálicas, produzindo um campo elétrico uniforme e constante
orientado para cima. Como as placas metálicas são muito grandes, toda a
região de oscilação do pêndulo é abrangida pelo campo elétrico
uniforme, fazendo com que o pêndulo passe a oscilar com um período 5T.
Nessas condições, a intensidade do campo elétrico uniforme e constante
estabelecido entre as placas metálicas vale

(A) 125 N/C (B) 150 N/C (C) 200 N/C (D) 240 N/C


Um abraço!

Luiz

-- 
Esta mensagem foi verificada pelo sistema de antiv�rus e
 acredita-se estar livre de perigo.



[obm-l] Re: [obm-l] [obm-l] Questão Geometria

2016-10-10 Por tôpico Carlos Victor
 

Olá Vinicius, 

Seja R a intersecção de AO com BC. Seja T a intersecção da bissetriz de
 Será que alguém poria me ajudar na seguinte questão? 
> 
> * 
> 
> (Belarus) Seja O o centro do círculo ex-inscrito do triângulo ABC oposto ao 
> vértice A. Seja M o ponto médio de AC e seja P a intersec ̧ão das retas MO e 
> BC. Prove que se ∠BAC = 2∠ACB, então AB = BP. 
> 
> -- 
> Esta mensagem foi verificada pelo sistema de antivrus e 
> acredita-se estar livre de perigo.
 
-- 
Esta mensagem foi verificada pelo sistema de antiv�rus e
 acredita-se estar livre de perigo.



[obm-l] Re: [obm-l] Re: [obm-l] Re: [obm-l] [obm-l] Questão Geometria

2016-10-09 Por tôpico Jeferson Almir
Sei que o tópico não tem nada a ver com o problema proposto, mas já postei
2 problemas que não aparecem na caixa da lista e percebi que alguns
receberam pois até responderam. Isso já aconteceu com alguém???

Em 9 de outubro de 2016 15:23, Israel Meireles Chrisostomo <
israelmchrisost...@gmail.com> escreveu:

> Se vc não quiser receber mais emails da obm l envie um emeail para obm l
>
> Em 8 de outubro de 2016 13:15, Matheus Herculano <
> matheusherculan...@gmail.com> escreveu:
>
>> A resposta é para de me mandar isso
>>
>> Em 1 de out de 2016 20:00, "vinicius raimundo" <vini.raimu...@gmail.com>
>> escreveu:
>>
>>> Será que alguém poria me ajudar na seguinte questão?
>>>
>>>
>>>1.
>>>
>>>(Belarus) Seja O o centro do círculo ex-inscrito do triângulo ABC oposto
>>>ao vértice A. Seja M o ponto médio de AC e seja P a intersec ̧ão das
>>>retas MO e BC. Prove que se ∠BAC = 2∠ACB, então  AB = BP.
>>>
>>>
>>> --
>>> Esta mensagem foi verificada pelo sistema de antivírus e
>>> acredita-se estar livre de perigo.
>>
>>
>> --
>> Esta mensagem foi verificada pelo sistema de antivírus e
>> acredita-se estar livre de perigo.
>>
>
>
> --
> Esta mensagem foi verificada pelo sistema de antivírus e
> acredita-se estar livre de perigo.
>

-- 
Esta mensagem foi verificada pelo sistema de antiv�rus e
 acredita-se estar livre de perigo.



[obm-l] Re: [obm-l] Re: [obm-l] Re: [obm-l] [obm-l] Questão Geometria

2016-10-09 Por tôpico Jeferson Almir
Perdão foi processado sim na Mail Archive acabo de constatar mas demorou
alguns dias para aparecer. Valeu!!

Em 9 de outubro de 2016 17:40, Jeferson Almir <jefersonram...@gmail.com>
escreveu:

> Sei que o tópico não tem nada a ver com o problema proposto, mas já postei
> 2 problemas que não aparecem na caixa da lista e percebi que alguns
> receberam pois até responderam. Isso já aconteceu com alguém???
>
> Em 9 de outubro de 2016 15:23, Israel Meireles Chrisostomo <
> israelmchrisost...@gmail.com> escreveu:
>
>> Se vc não quiser receber mais emails da obm l envie um emeail para obm l
>>
>> Em 8 de outubro de 2016 13:15, Matheus Herculano <
>> matheusherculan...@gmail.com> escreveu:
>>
>>> A resposta é para de me mandar isso
>>>
>>> Em 1 de out de 2016 20:00, "vinicius raimundo" <vini.raimu...@gmail.com>
>>> escreveu:
>>>
>>>> Será que alguém poria me ajudar na seguinte questão?
>>>>
>>>>
>>>>1.
>>>>
>>>>(Belarus) Seja O o centro do círculo ex-inscrito do triângulo ABC oposto
>>>>ao vértice A. Seja M o ponto médio de AC e seja P a intersec ̧ão
>>>>das retas MO e BC. Prove que se ∠BAC = 2∠ACB, então  AB = BP.
>>>>
>>>>
>>>> --
>>>> Esta mensagem foi verificada pelo sistema de antivírus e
>>>> acredita-se estar livre de perigo.
>>>
>>>
>>> --
>>> Esta mensagem foi verificada pelo sistema de antivírus e
>>> acredita-se estar livre de perigo.
>>>
>>
>>
>> --
>> Esta mensagem foi verificada pelo sistema de antivírus e
>> acredita-se estar livre de perigo.
>>
>
>

-- 
Esta mensagem foi verificada pelo sistema de antiv�rus e
 acredita-se estar livre de perigo.



[obm-l] Re: [obm-l] Re: [obm-l] [obm-l] Questão Geometria

2016-10-09 Por tôpico Israel Meireles Chrisostomo
Se vc não quiser receber mais emails da obm l envie um emeail para obm l

Em 8 de outubro de 2016 13:15, Matheus Herculano <
matheusherculan...@gmail.com> escreveu:

> A resposta é para de me mandar isso
>
> Em 1 de out de 2016 20:00, "vinicius raimundo" <vini.raimu...@gmail.com>
> escreveu:
>
>> Será que alguém poria me ajudar na seguinte questão?
>>
>>
>>1.
>>
>>(Belarus) Seja O o centro do círculo ex-inscrito do triângulo ABC oposto
>>ao vértice A. Seja M o ponto médio de AC e seja P a intersec ̧ão das
>>retas MO e BC. Prove que se ∠BAC = 2∠ACB, então  AB = BP.
>>
>>
>> --
>> Esta mensagem foi verificada pelo sistema de antivírus e
>> acredita-se estar livre de perigo.
>
>
> --
> Esta mensagem foi verificada pelo sistema de antivírus e
> acredita-se estar livre de perigo.
>

-- 
Esta mensagem foi verificada pelo sistema de antiv�rus e
 acredita-se estar livre de perigo.



[obm-l] Re: [obm-l] [obm-l] Questão Geometria

2016-10-08 Por tôpico Matheus Herculano
A resposta é para de me mandar isso

Em 1 de out de 2016 20:00, "vinicius raimundo" <vini.raimu...@gmail.com>
escreveu:

> Será que alguém poria me ajudar na seguinte questão?
>
>
>1.
>
>(Belarus) Seja O o centro do círculo ex-inscrito do triângulo ABC oposto
>ao vértice A. Seja M o ponto médio de AC e seja P a intersec ̧ão das
>retas MO e BC. Prove que se ∠BAC = 2∠ACB, então  AB = BP.
>
>
> --
> Esta mensagem foi verificada pelo sistema de antivírus e
> acredita-se estar livre de perigo.

-- 
Esta mensagem foi verificada pelo sistema de antiv�rus e
 acredita-se estar livre de perigo.



  1   2   3   4   5   6   7   8   9   10   >